Logical Reasoning: Examples Part 2

Pataasin ang iyong marka sa homework at exams ngayon gamit ang Quizwiz!

An instructor presented two paintings to a class. She said that the first had hung in prestigious museums but the second was produced by an unknown amateur. Each student was asked which painting was better. Everyone selected the first. The instructor later presented the same two paintings in the same order to a different class. This time she said that the first was produced by an unknown amateur but the second had hung in prestigious museums. In this class, everyone said the second painting was better. The statements above, if true, most strongly support which one of the following? C. Each of the students would like most of the paintings hanging in any prestigious museum. D. In judging the paintings, some of the students were affected by what they had been told about the history of the paintings.

C. Incorrect. "Most" is unsupported. Furthermore, we only know about preference between two paintings, not about liking overall. D. Correct. History of being produced by an unknown amateur or being hung in a prestigious museum influenced preference.

Milk question

D. Incorrect. Addresses taste but not enzyme E. Correct. In this case, it would be the excess heat causing the destruction and not the microwaves. The microwaves were the means to the heat. This resolves the apparent discrepancy and would weaken the argument by illustrating the possibility that heat was the cause of the enzyme reduction and not the microwave like the conclusion dictates.

It is now a common complaint that the electronic media have corroded the intellectual skills required and fostered by the literary media. But several centuries ago the complaint was that certain intellectual skills, such as the powerful memory and extemporaneous eloquence that were intrinsic to oral culture, were being destroyed by the spread of literacy. So, what awaits us is probably a mere alteration of the human mind rather than its devolution. The reference to the complaint of several centuries ago that powerful memory and extemporaneous eloquence were being destroyed plays which one of the following roles in the argument? A. Evidence supporting the claim that the intellectual skills fostered by the literary media are being destroyed by the electronic media. B. An illustration of the general hypothesis being advanced that intellectual abilities were inseparable from the means by which people communicate. C. An example of a cultural change that did not necessarily have a detrimental effect on the human mind overall. D. Evidence that the claim that the intellectual skills required and fostered by the literary media are being lost is unwarranted. E. Possible evidence, mentioned and then dismissed, that might be cited by supporters of the hypothesis being criticized.

DETERMINE THE FUNCTION Arguer: Quasi-Opposing Point: People complain about electronic media killing literary skills. Premise: But people complained about literacy killing oral culture skills. Conclusion: Electronic media results are likely a change in the mind, not devolution of the mind. Opponent: Opponent's Premise: Electronic media is killing literary skills. Opponent's Conclusion: Therefore, our brains are going to devolve! The claim is used as support for the conclusion, since it is an example of a previous and similar change in (not devolution of) the human mind. A. Incorrect. Does not support the Quasi-Opposing Point but rather the author's conclusion. B. Incorrect. Hypothesis is out of scope. C. Correct. D. Incorrect. The author's conclusion is weak ("probably") and never describes the opposition's claim as "unwarranted." Furthermore, the author never disputes the claim/complaint that "the intellectual skills required and fostered by the literary media are being lost." Rather, the author disputes the conclusion supported by this claim that "our brains are going to devolve." E. Incorrect. Describes the "complaint of several centuries ago that powerful memory and extemporaneous eloquence were being destroyed" as support for opponents rather than support for the author. Remember that this sentence functions as a premise for the author's conclusion. Furthermore, the author never "dismisses" this claim.

Public health expert: until recently, people believed that the application of biochemical research would eventually achieve complete victory over the microorganisms that cause human disease. However, current medical research shows that those microorganisms reproduce so rapidly that medicines developed for killing one variety will only spur the evolution of other varieties that are immune to those medicines. The most rational public health strategy, therefore, would place much more emphasis than at present on fully informing people about the transmission of diseases caused by microorganisms, with a view to minimizing the incidence of such diseases. C. There is good reason to make a particular change to public health policy. E. Some previous approaches to public health policy ignored the fact that disease-causing microorganisms reproduce at a rapid rate.

DETERMINE THE FUNCTION P1: previous strategy of biochem P2: new research of evolution to medication C: new public health strategy on informing people about the transmission of diseases C. Correct. E. Incorrect. An implied premise. Does not evoke conclusion of adopting a new strategy

Ethicist: it would be a mistake to say that just because someone is not inclined to do otherwise, she or he does not deserve to be praised for doing what is right, for although we do consider people especially virtuous if they successfully resist a desire to do what is wrong, they are certainly no less virtuous if they have succeeded in extinguishing all such desires. The assertion that people are considered especially virtuous if they successfully resist a desire to do what is wrong plays which one of the following roles in the ethicist's argument? B. It makes an observation that, according to the argument, is insufficient to justify the claim that the argument concludes is false. C. It is a claim, acceptance of which, the argument contains, is a primary obstacle to some people's having an adequate conception of virtue. D. It is, according to the argument, a commonly held opinion that is nevertheless false.

DETERMINE THE FUNCTION P: although we do consider people especially virtuous if they resist the desire to do wrong... P2: no less virtuous if they do not have those desires. C: it is false that someone should not be praised for an act that they would do anyways B. Correct. Pay attention to the "although". C. Incorrect. "Primary obstacle" is out of scope. D. Incorrect. Author does not state that this claim is false.

The temperature in T is always at least 10 degrees lower than the temperature in C. However, the average resident of C spends 10 to 20 percent more on winter heating expenses than does the average resident of T. Each of the following, if true, helps to resolve the apparent paradox described above EXCEPT: A. Heat loss due to wind is less in T than in C. B. Although C is always fairly warm during the daytime, temperatures in C drop steeply at night. C. Utility rates in T are lower than utility rates in C. D. People who are used to warmer temperatures generally keep their homes warmer in the winter than do people who are used to colder temperatures. E. Houses in colder climates are usually better insulated than houses in warmer climates.

EXPLAIN A RESULT A, C, D, E. Incorrect. All explain the discrepancy. B. Correct. Does not change the fact that T is ALWAYS 10 degrees colder; this pattern should persist even at night

Shark teeth are among the most common vertebrate fossils; yet fossilized shark skeletons are much less common--indeed, comparatively rare among fossilized vertebrate skeletons. Which one of the following, if true, most helps to resolve the apparent paradox described above? A. Unlike the bony skeletons of other vertebrates, shark skeletons are composed of cartilage, and teeth and bone are much more likely to fossilize than cartilage is. C. Fossils of sharks' teeth are quite difficult to distinguish from fossils of other kinds of teeth. D. Some species of sharks alive today grow and lose many sets of teeth during their lifetimes.

EXPLAIN A RESULT A. Correct. Shark skeleton cartilage fossilize less frequently than the skeleton bones of other vertebrates. Shark teeth fossilize at the same rate as bone. C. Incorrect. Undermines the premise. Implies that there may be a mixup about shark teeth fossils, and shark teeth fossils are not actually that common. Does not explain why fossilized shark skeletons are less common than fossilized vertebrate skeletons. D. Incorrect. Does not explain why fossilized shark skeletons are less common than fossilized vertebrate skeletons. Furthermore, losing teeth doesn't necessarily mean the teeth are more likely to be fossilized. Finally, "some" is not quite strong enough. Furthermore, the subject of sharks living today is out of scope; we are concerned about fossilized shark teeth.

Automated flight technology can guide an aircraft very reliably, from navigation to landing. Yet this technology, even when functioning correctly, is not a perfect safeguard against human error. Which one of the following, if true, most helps to explain the situation above? B. Smaller aircraft do not always have their automated flight technology updated regularly. C. If a plane's automated flight technology malfunctions, crew members have to operate the plane manually. E. Automated flight technology invariably executes exactly the commands that humans give it.

EXPLAIN A RESULT Make note of the limitation "when functioning correctly" in the conclusion. B. Incorrect. The conclusion controls for this with the limitation "when functioning correctly". C. Incorrect. ^ E. Correct. Technology represents all human errors and accomplishments.

Heart attacks are most likely to occur on Mondays. The accepted explanation is that because Monday is the first day of the workweek, people feel more stress on Mondays than on other days. However, research shows that even unemployed retired people are more likely to have heart attacks on Mondays than on other days. Which one of the following, if true, most helps to explain the increased likelihood that an unemployed retiree will have a heart attack on a Monday? A. Because they associate Monday with work, retired people are more likely to begin large projects on Mondays. B. Many retired people take up part-time jobs after they retire from their careers. C. People seldom change their dietary and other health habits after retirement

EXPLAIN A RESULT P: unemployed retirees are also more prone to heart attacks on Mondays. C: there must be some reason other than work stress that makes Monday heart attack day. A: unemployed retirees do not still feel the stress associated with going back to work on Monday. A. Correct. Questions assumption. Unemployed retirees may still experience work-related stress on Mondays. B. Incorrect. Answer choice deals with employed retired people, yet we are concerned with unemployed retired people. C. Incorrect. Diet and health are out of scope/unrelated to stress

No one with a serious medical problem would rely on the average person to prescribe treatment. Similarly, since a good public servant has the interest of the public at heart, ___. Which one would most reasonably complete the argument D. Public servants should base decisions on something over than the average person's recommendation. E. One is a good public servant if one is more knowledgable about the public good than is the average person.

FILL IN THE BLANK Complete the analogy. D. Correct. Tempting to eliminate, since this answer choice does not indicate that the public servant should accept recommendations from an above average person. However, notice that NEITHER DOES THE STIMULUS in reference to medical recommendations. E. Incorrect. Stimulus does not mention a person's knowledge or what it means to be a "good public servant"; it's all about a person being responsible in whom they take recommendations from.

Hospital executive: At a recent conference on nonprofit management, several computer experts maintained that the most significant threat faced by large institutions such as universities and hospitals is unauthorized access to confidential data. In light of this testimony, we should make the protection of our clients' confidentiality our highest priority. The hospital executive's argument is most vulnerable to which one of the following objections? A. The argument confuses the causes of a problem with the appropriate solutions to that problem. B. The argument relies on the testimony of experts whose expertise is not shown to be sufficiently broad to support their general claim. C. The argument assumes that a correlation between two phenomena as evidence that one is the cause of the other. D. The argument draws a general conclusion about a group based on data about an unrepresentative sample of that group. E. The argument infers that a property belonging to large institutions belongs to all institutions.

FLAW A. Incorrect. Does not occur. B. Correct. The hospital executive uses the testimony of computer experts (with a limited set of expertise) in order to make a conclusion about what the hospital's main priority should be (a very broad strategic decision). C. Incorrect. Does not occur. Does not confuse/address correlation. D. Incorrect. No sampling flaw. E. Incorrect. Does not infer beyond the hospital.

On the basis of the available evidence, Antarctica has generally been thought to have been covered by ice for at least the past 14 million years. Recently, however, three-million-year-old fossils of a kind previously found only in ocean-floor sediments were discovered under the ice sheet covering central Antarctica. About three million years ago, therefore, the Antarctic ice sheet must temporarily have melted. After all, either severe climatic warming or volcanic activity in Antarctica's mountains could have melted the ice sheet, thus raising sea levels and submerging the continent. The reasoning in the argument is most vulnerable to which one of the following criticisms? B. That either of two things could independently have produced a given effect is taken to show that those two things. E. An inconsistency that, as presented, has more than one possible resolution is treated as though only one resolution is possible.

FLAW B. Incorrect. The argument never states that climatic warming or volcanic activity occurred independently. E. Correct. The argument assumes that those fossils were developed because the ice sheet melted, raised sea levels, and submerged the continent, rather than considering other possible resolutions

Eighteenth-century European aesthetics was reasonably successful in providing an understanding of all art, including early abstract art, until the 1960s, when artists self-consciously rebelled against earlier notions of art. Since the work of these rebellious artists is quite beautiful but outside the bounds of the aesthetic theory then current, there can be no complete theory of aesthetics. The reasoning above is most vulnerable to criticism in that it B. Presumes, without providing justification, that artists' rebellion in the 1960s against earlier notions of art was not guided by their knowledge of eighteenth-century European aesthetic theory D. Presumes, without providing justification, that art from the 1960s is the only art that cannot be adequately addressed by eighteenth-century European aesthetics E. Presumes, without providing justification, that eighteenth-century European aesthetics is as encompassing as an aesthetic theory can be

FLAW P1: Reasonably complete theory of aesthetics in the 1700s P2: Art in the 1960s falls outside of that theory, as well as outside of the aesthetics theory in the 1960s. C: There can be no complete theory of aesthetics. A: There are no other theories more complete than the 18th-c European theory (ignores other potential theories of aesthetics) A: Since there was ONCE a style of art that fell outside current aesthetic theory, there will ALWAYS be some new art form that eludes the boundaries of the then-current aesthetic theory B. Incorrect. Knowledge of theory does not change the fact that 1960s art falls outside of the 1700 theories C. Incorrect. Does not occur. 1960s art is the only style of art mentioned that fell outside 18th century European aesthetics, but that doesn't mean the author has to assume that ONLY 1960s art falls outside. It's possible that art in the 70's, 80's, 90's, etc. ALSO falls outside. E. Correct. States that we will never attain a complete theory by arguing, "We basically HAD a complete theory, but then a form of art came along and didn't fit inside. Thus, we're forever screwed."

One is likely to feel comfortable approaching a stranger if the stranger is of one's approximate age. Therefore, long-term friends are probably of the same approximate age as each other since most long-term friendships begin because someone felt comfortable approaching a stranger. The reasoning in the argument is flawed in that it A. Presumes, without warrant, that one is likely to feel uncomfortable approaching a person only if that person is a stranger. B. Infers that a characteristic is present in a situation from the fact that that characteristic is present in most similar situations. C. Overlooks the possibility that one is less likely to feel comfortable approaching someone who is one's approximate age if that person is a stranger than if that person is not a stranger. D. Presumes, without warrant, that one never approaches a stranger unless one feels comfortable doing so. E. Fails to address whether one is likely to feel comfortable approaching a stranger who is not one's approximate age.

FLAW P1: Same age --> comfortable approach. P2: Comfortable approach --> long-term friends. C: Long-term friends = same age. Assumption: Comfortable approach --> same age. A. Incorrect. -Comfortable approach --> stranger does not occur. Furthermore, it fails the negation test. One could feel uncomfortable approaching a person who was not a stranger and the reasoning would not be affected. B. Incorrect. Too extreme; the stimulus uses "probably." C. Incorrect. Comparison between strangers and non-strangers does not occur and is not relevant. D. Incorrect. Does not occur. The stem only talks about when people feel comfortable approaching strangers, not when they feel uncomfortable. E. Correct. Does not establish -same age --> -comfortable approach in order to justify the illegal reversal comfortable approach --> same age.

Bardis: extensive research that television advertisements affect the buying habits of consumers. Some people conclude from this that violent television imagery sometimes causes violent behavior. But the effectiveness of television advertisements could be a result of those televised images being specifically designed to alter buying habits, whereas television violence is not designed to cause violent behavior. Hence we can safely conclude that violent television imagery does not cause violence. The reasoning in Bardis's argument is flawed because that argument A. Relies on an illegitimate inference from the fact that advertisements can change behavior to the claim that advertisements can cause violent behavior. B. Fails to distinguish a type of behavior from a type of stimulus that may or may not affect behavior. D. Concludes that a claim is false on the basis of one purported fault in an argument in favor of that claim.

FLAW P1: although television advertisements affect the buying habits of consumers, they are designed to do so. P2: violent television is not designed to cause violent behavior C: violent television imagery does not cause violence. A. Incorrect. This inference opposes the conclusion that advertisements do not cause violent behavior. B. Incorrect. Does not occur; distinguishes ads and violent TV from purchasing behavior and violent behavior. D. Correct. Concludes that the claim "violent television imagery sometimes causes violent behavior" is false upon observing that within supporting evidence, televised images, unlike violent television, are specifically designed to alter buying habits.

Doctor: medical researchers recently examined a large group of individuals who said that they had never experienced serious back pain. Half of the members of the group turned out to have bulging or slipped disks in their spines, conditions often blamed for serious back pain. Since these individuals with bulging or slipped disks evidently felt no pain from them, these conditions could not lead to serious back pain in people who do experience such pain. The reasoning in the doctor's argument is most vulnerable to the criticism that it fails to consider which one of the following possibilities? B. A factor that is not in itself sufficient to produce a certain effect may nonetheless be partly responsible for that effect in some instances. E. A factor that does not bring about a certain effect may nonetheless be more likely to be present when the effect occurs than when the effect does not occur.

FLAW P: -Pain Bulging or slipped disks C: Bulging or slipped disks --> -pain B. Correct. Just because bulging or slipped disks are not sufficient to/do not always produce pain does not mean that they cannot produce pain. E. Incorrect. The conclusion does not discuss the likelihood of bulging or slipped disks being present, but whether that factor actually contributes to the effect of back pain.

Columnist: neuroscientists have found that states of profound creativity are accompanied by an increase of theta brain waves, which occur in many regions of the brain, including the hippocampus. They also found that listening to music increases theta waves dramatically. Thus, one can attain a state of profound creativity by listening to a tape of recorded music. The columnist's reasoning is most vulnerable to criticism on the grounds that it A. Takes for granted that there is a causal connection between the hippocampus and being in a state of creativity. B. Fails to consider that music is not necessary for one to be in a state of profound creativity. C. Does not rule out the possibility that to music by means other than a tape recording also increase theta waves. D. Ignores the possibility that an increase in theta waves may not always be accompanied by a state of profound creativity. E. Provides insufficient reasons to believe that people who are not in states of profound creativity have low levels of theta brain waves.

FLAW P: Creativity and theta waves correlated. Music increases theta waves. C: Music can increase theta waves and thus creativity. A. Incorrect. Does not make assumptions based on hippocampus specifically. B. Incorrect. Does not commit this flaw. Instead, erroneously assumes that music is sufficient, rather than necessary, for creativity. C. Incorrect. Does not commit this flaw. Describes music as one cause of creativity, not the only cause. No bearing on conclusion. D. Correct. Theta waves are correlated with but do not guarantee creativity. The author erroneously assumes that theta waves —> creativity. E. Incorrect. Does not commit this flaw. Negates the premise. We do not know about those not in states of profound creativity

Although high cholesterol levels have been associated with the development of heart disease, many people with high cholesterol never develop heart disease, while many without high cholesterol do. Recently, above average concentrations of the blood particle lipoprotein(a) were found in the blood of many people whose heart disease was not attributable to other causes. Dietary changes that affect cholesterol levels have no effect on lipoprotein(a) levels. Hence, there is no reason for anyone to make dietary changes for the same of preventing heart disease. Which one of the following most accurately describes a flaw in the argument? A. It fails to consider the possibility that lipoprotein(a) raises cholesterol levels. C. It presents but ignores evidence that, for some people, high cholesterol contributes to heart disease.

FLAW P: (1) high cholesterol levels have been associated with high risk for heart disease, yet some with high cholesterol develop heart disease and some with high cholesterol do not. (2) some with high lipoprotein(a) develop heart disease not attributable to other causes. (3) dietary changes that affect cholesterol do not effect lipoprotein(a) levels. C: no reason to make dietary changes to prevent heart disease. Flaw: uses evidence based on heart disease developed from lipoprotein to conclude about heart disease in general. Assumes that cholesterol (and thus dietary changes that influence cholesterol) has no impact A. Incorrect. Does not occur. Stimulus considers heart disease "not attributed to other causes" besides lipoprotein(a); thus, lipoprotein(a) must not increase cholesterol levels to cause heart disease. C. Correct. Cannot assume that cholesterol is never a cause of heart disease; ignores all ambiguous evidence presented.

Pundit: people complain about how ineffectual their legislative representatives are, but this apparent ineffectuality is simply the manifestation of compromises these representatives must make when they do what they were elected to do: compete for the government's scarce funds. So, when people express dissatisfaction with their legislative representatives, we can be assured that these representatives are simply doing what they were elected to do. The pundit's argument is flawed because it takes for granted that A. The apparent ineffectuality of legislative representatives is the only source of popular dissatisfaction with those representatives. D. Legislative compromise inevitably results in popular dissatisfaction with politicians

FLAW P: Ineffectuality --> compromise --> compete for funds C: Public dissatisfaction --> representatives doing what they were elected to do A: Public dissatisfaction --> ineffectuality --> compromise --> compete for funds --> representatives doing what they were elected to do A. Correct Public dissatisfaction --> ineffectuality D. Incorrect Compromise --> popular dissatisfaction -Popular dissatisfaction --> -compromise Illegal negation

Concert promoter: some critics claim that our concert series lacks popular appeal. But our income from the sales of t-shirts and other memorabilia at the concerts is equal to or greater than that for similar sales at comparable series. The concert promoter's argument is flawed in that it B. Takes for granted that income from sales of memorabilia is the sole indicator of popular appeal. C. Takes for granted that the comparable series posses popular appeal.

FLAW P: income from this series is equal or greater than income from other series C: concert series has popular appeal A: the other series in the comparison have popular appeal B. Incorrect. Assumes that income is AN indicator, but not necessarily the SOLE indicator. C. Correct. Address the assumption.

Computer profits ? Screenshotted The reasoning in the manager's argument is vulnerable to criticism on which of the following grounds? A. The argument fails to consider the possibility that the money earned on each high-end computer is significantly higher than the money earned on each low-end computer. C. The argument ignores the possibility that some customers who come into a computer store expecting to purchase a low-end model end up purchasing a high-end model.

FLAW Premises: 1. Last year the store made an average profit of 13% on computers selling for > $1000 (high-end) and a typical profit of 25% or more on computers selling for <$1000 (low-end). 2. If the store sells only low-end models then they will probably sell as many computers as they will if they continue selling both low-end and high-end models. Intermediate Conclusion: Selling only low-end models will maximize profits. Main Conclusion: The store should only sell low-end models. We don't know the exact price for each type of computer. The high-end models could sell for much more than $1000, and the low-end for much less. Suppose the low-end computers sell for $10 each, and the high-end for $10,000. The store would make $1,300 on each high-end computer, and $2.50 on each low-end model. Now, selling only low-end models does not sound good idea? A. Correct. C. Incorrect. Does not adequately indicate a flaw. Again, without knowing the price of each model, we don't know if this has any impact.

The presence of bees is necessary for excellent pollination, which, in turn usually results in abundant fruits and vegetables. Establishing a beehive or two near one's garden ensures the presence of bees. Keeping bees is economical, however, only if the gardener has a use for homegrown honey. Thus, gardeners who have no use for homegrown honey will tend not to have beehives, so their gardens will ail to have excellent pollination. Which one of the following most accurately describes a flaw in the reasoning of the argument? A. The argument fails to consider the possibility that obtaining homegrown honey is only one of several advantages of beehives. B. The argument confuses what is necessary for pollination to take place with what would guarantee that it takes place. D. The argument fails to consider that bees might be present even in the absence of a particular condition that would ensure their present.

FLAW Reasoning: Excellent pollination --> bees Beehive --> bees Economic benefit --> use for honey -Use for honey --> -Beehive -Beehives --> -excellent pollination Assumption: -Beehives --> -bees --> -excellent pollination While we can accept that -bees --> -excellent pollination, we cannot accept that -beehives --> -bees; this is illegal reversal. A. Incorrect. Does not dispute the conclusion that gardeners without a use for honey will not experience excellent pollination. B. Incorrect. Engages in illegal reversal not for "excellent pollination --> bees" but for "beehives --> bees." D. Correct. Assumes that since beehives ensures bees, -beehives ensures -bees. Illegal reversal.

Many economists claim that financial rewards provide the strongest incentive for people to choose one job over another. But in many surveys, most people do not name high salary as the most desirable feature of a job. Which one of the following, if true, most weakens the argument? B. In many surveys, people say that they would prefer a high-wage job to an otherwise identical job with lower wages. C. Jobs that pay the same salary often vary considerably in their other financial benefits.

FLAW The argument assumes that if people were motivated most by money in job choices, they would name high salary as the most desirable feature. A flaw in this reasoning is that salary is the only form of motivation by money. B. Incorrect. Demonstrates that people compare high wages to low wages, but does not relate money to job considerations. C. Correct. Suggests that individuals may be motivated by money in ways other than salary.

Although human economic exchange predates historical records, it is clear that the very first economies were based on barter and that money came later. This can be inferred from occasions in history when, in isolated places, currency largely disappeared from the local economy. At such times, the economy typically reverts to the original barter system, but then quickly abandons this form of exchange when currency becomes available again. Which one of the following describes a flaw in the argument's reasoning? B. The argument contains premises that contradict one another. E. The argument relies on a premise that presupposes that the argument attempts to show in the conclusion.

FLAW P: Examples where in isolated places, the currency disappears and barter typically returns (whereas they go back to currency once currency is available again). C: First economies based on barter; money came later. B. Incorrect. C: barter came first, then money. P: currency is there, then barter, then currency again. P undermines C. P potentially undermines the statement in C that "although human economic exchange predates historical records." However, there are no contradictions amongst the PREMISES. E. Correct. Circular reason because (1) P does not prove anything about the very first economies and (2) the author is only makes P relevant by claiming that they "REVERT to the ORIGINAL barter system". How do we know that they were ORIGINALLY a barter system? This presumption is based on what the author is hoping to prove in C.

Some twentieth-century art is great art. All great art involves original ideas, and any art that is not influential cannot be great art. Each of the following statements follows logically from the set of statements above EXCEPT: A. Some influential art involves original ideas. B. Some twentieth-century art involves original ideas. C. Only art that involves original ideas is influential. D. Only art that is influential and involves original ideas is great art. E. Some twentieth-century art is influential and involves original ideas.

Given: Twentieth-century art -(some)-> great art. Great art --> original ideas. -Influential --> -great art. Inference: Twentieth-century art -(some)-> great art --> original ideas and influential. A. Incorrect. This is inferable. We know that some twentieth-century art is both original ideas and influential. Thus some influent art must involve original ideas. B. Incorrect. This is inferable. C. Correct. Influential --> original is not inferable. D. Incorrect. This is inferable. Great art --> original ideas and influential. E. Incorrect. This is inferable.

On the basis of the available evidence, Antarctica has generally been thought to have been covered by ice for at least the past 14 million years. Recently, however, three-million-year-old fossils of a kind previously found only in ocean-floor sediments were discovered under the ice sheet covering central Antarctica. About three million years ago, therefore, the Antarctic ice sheet must temporarily have melted. After all, either severe climatic warming or volcanic activity in Antarctica's mountains could have melted the ice sheet, thus raising sea levels and submerging the continent. Which one of the following is the main conclusion of the argument? A. Antarctica is no longer generally thought to have been covered by ice for the past 14 million years. C. The ice sheet covering Antarctica has not been continuous present throughout the past 14 million years. E. The ice sheet covering Antarctica was melted either as a result of volcanic activity in Antarctica's mountains or as a result of severe climatic warming.

ID THE CONCLUSION P: 3-million-year-old fossils from ocean-floor sentiments discovered under ice sheet Supporting background information: climate warming or volcanic activity could have melted ice sheet to raise sea levels and submerge content C: ice sheet temporarily melted 3 million years ago A. Incorrect. "Generally thought" is unsupported; we only know what the author of this argument thinks. C. Correct. E. Incorrect. Offers supporting background information

A distinguished British judge, Justice Upton, said that whether some administrative decision by a government minister is reasonable "is a question that judges, by their training and experience, should be well-equipped to answer, or else there would be something badly wrong with the legal system, and there is little reason to suppose that there is." Which one of the following most accurately expresses the conclusion o Justice Upton's argument? A. There is nothing much wrong with the legal system. B. Judges should be given a greater part in administrative decision making. C. Judges are qualified to decide upon the reasonableness of a government minister's administrative decision. D. If something were badly wrong with the legal system, judges would be ill-equipped to determine whether a government's minister's decisions are reasonable. E. If judges are well-equipped to determine whether an administrative decision is reasonable, there is not anything badly wrong with the legal system.

IDENTIFY THE CONCLUSION -Equipped --> wrong with legal system -Wrong with legal system Equipped "Administrative decision...is a question that judges...should be well-equipped to answer." A. Incorrect. Premise to justify that judges should make administrative decisions. B. Incorrect. Out of scope. C. Correct. D. Incorrect. Wrong --> -equipped. Illegal reversal. E. Incorrect. Equipped --> -wrong. Illegal negation.

A ring of gas emitting X-rays flickering 450 times per second has been observed in a stable orbit around a black hole. In light of certain widely accepted physical theories, that rate of flickering can best be explained if the ring of gas has a radius of 49 kilometers. But the gas ring could not maintain an orbit so close to a black hole unless the black hole was spinning. The statements above, if true, most strongly support which one of the following, assuming that the widely accepted physical theories referred to above are correct? A. Black holes that have orbiting rings of gas with radii greater than 49 kilometers are usually stationary. B. Only rings of gas that are in stable orbits around black holes emit flickering X-rays. C. The black hole that is within the ring of gas observed by the astronomers is spinning. D. X-rays emitted by rings of gas orbiting black holes cause those black holes to spin. E. A black hole is stationary only if it is orbited by a ring of gas with a radius of more than 49 kilometers.

INFERENCE 1. Ring of gas emitting X-rays flickering 450 times per second. 2. Stable orbit around black hole. 3. Ring of gas with radius of 49 kilometers best explains ring of gas emitting X-rays flickering 450 times per second. 4. -Black hole spinning --> -ring of gas with radius of 49 kilometers. Inferences... Since ring of gas with radius of 49 kilometers best explains ring of gas emitting X-rays flickering 450 times per second, ring of gas with radius of 49 kilometers --> black hole spinning. A. Incorrect. Illegal reversal. A claims -ring of gas with radius of 49 kilometers --> -black hole spinning, when it is given that -black hole spinning --> -ring of gas with radius of 49 kilometers. Furthermore, detail creep; we know nothing about "greater than 49 kilometers" specifically and "stationary" is not necessarily the same as "-black hole spinning." B. Incorrect. X-rays flickering --> stable orbits is unsupported. C. Correct. X-ray flickering suggests radius of 49 --> black hole spinning. D. Incorrect. X-rays flickering --> black hole spinning is unsupported. E. Incorrect. Ring of gas with radius of more than 49 kilometers --> black hole stationary. Detail creep; "ring of gas with radius of more than 49 kilometers" is not the same as "-ring of gas with radius of 49 kilometers" and "stationary" is not the same thing as "-spinning." Even then, this answer choice would imply -ring of gas with radius of 49 kilometers --> -black hole spinning, which is illegal negation.

Modern science is built on the process of posing hypotheses and testing them against observations--in essence, attempting to show that the hypotheses are incorrect. Nothing brings more recognition than overthrowing conventional wisdom. It is accordingly unsurprising that some scientists are skeptical of the widely accepted predictions of global warming. What instead remarkable is that with hundreds of researchers striving to make breakthroughs in climatology, very few find evidence that global warming is unlikely. The information above provides the most support for which one of the following statements? B. Most researchers in climatology have substantial motive to find evidence that would discredit the global warming hypothesis. E. Research in global warming is primarily driven by a desire for recognition in the scientific community.

INFERENCE 1. Science aims to overthrow conventional wisdom. 2. Unsurprising that climate science is skeptical of conventional predictions of global warming. 3. Surprising that climate scientists have not found evidence contrary to global warming. B. Correct. The global warming hypothesis is conventional wisdom that science often aims to overthrow. E. "Primarily driven" is out of scope.

The solidity of bridge piers built on pilings depends largely on how deep the pilings are driven. Prior to 1700, pilings were driven to "refusal", that is, to the point at which they refused to go any deeper. In a 1588 inquiry into the solidity of piers for Venice's Rialto Bridge, it was determined that the bridge's builder, Antonio Da Ponte, had met the contemporary standard to refusal: he had caused the pilings to be driven until additional penetration into the ground was no greater than two inches after twenty-four hammer blows. Which one of the following can properly be inferred from the passage? C. Da Ponte's standard of refusal was less strict than that of other bridge builders of his day. E. It is possible that the pilings of the Rialto Bridge could have been driven deeper even after the standard of refusal had been met.

INFERENCE C. Incorrect. The stimulus tells us how pilings were driven to "refusal" but not about the STANDARD used by the other bridge builders. E. Correct. Given that Da Ponte met the contemporary standard. Given that the pilings could have been driven in deeper, although at less than two inches after twenty-four hammer blows.

Art historian: More than any other genre of representational painting, still-life painting lends itself naturally to art whose goal is the artist's self-expression, rather than merely the reflection of a preexisting external reality. This is because in still-life painting, the artist invariably chooses, modifies, and arranges the objects to be painted. Thus, the artist has considerably more control over the composition and subject of a still-life painting than over those of a landscape painting or portrait, for example. Which one of the following is most strongly supported by the art historian's statements? D. In genres of representational painting other than still-life painting, the artist does not always choose, modify, and arrange the objects to be painted. E. When painting a portrait, artists rarely attempt to express themselves through the choice, modification, or arrangement of the background elements against which the subject of the portrait is painted.

INFERENCE D. Correct. "Does not always" = "does not invariably," the opposite of "invariably." E. Incorrect. "Rare" is too extreme/not the opposite of "invariably." We only know that compared to artists of still-life, artists of portrait do NOT INVARIABLY choose, modifies, and arranges the objects to be painted. It is possible that the portrait artist chooses, modifies and arranges quite frequently not always for this argument to hold.

Historian: the standard "QWERTY" configuration of the keys on typewriters and computer keyboards was originally designed to be awkward and limit typing speed. This was because early typewriters would jam frequently if adjacent keys were struck in quick succession. Experiments have shown that keyboard configurations more efficient than QWERTY can double typing speed while tremendously reducing typing effort. However, the expense and inconvenience of switching to a new keyboard configuration prevent any configuration other than QWERTY from attaining widespread use. Which one of the following is most strongly supported by the historian's statements? A. Most people who have tried typing with non-QWERTY keyboards have typed significantly more quickly using those keyboards than they usually have done using QWERTY keyboards. E. If the keyboard had been designed for computers, then it would not have been designed to limit typing speed.

INFERENCE Example of an inference question where the correct answer cannot be perfectly inferred but is the most provable. "QWERTY" configuration --> awkward and limit speed --> limit jamming A. Incorrect. We know that some configurations allow for faster typing than the QWERTY configuration, but we cannot claim that most do. E. Correct. Since we are told QWERTY was designed to address a specific and relatively unique issue involving the use of one particular device (the typewriter), it's not too much of a stretch to say that a different device wouldn't require such a design characteristic.

Zack's Coffeehouse schedules free poetry readings almost every Wednesday. Zack's offers half-priced coffee all day every day that a poetry reading is scheduled. Which one of the following can be properly inferred from the information above? C. Free poetry readings are scheduled on almost every day that Zack's offers half-priced coffee all day. D. Zack's offers half-priced coffee all day on most if not all Wednesdays. E. On some Wednesdays Zack's does not offer half-priced coffee all day.

INFERENCE If Zack's offers half-price coffee all day on days that there are poetry readings, and there are poetry readings on almost every Wednesday, then Zack's offers half-price coffee all day on almost every Wednesday. Wednesday -(most)-> free poetry reading --> poetry reading --> half-priced coffee. C. Incorrect. Engages in reverse logic: all poetry reading days feature a coffee deal, but we can't conclude that all, or even most, coffee deal days feature poetry readings. Possible that Zack's offers half-priced coffee but no free poetry readings on Thursdays. D. Correct. E. Incorrect. Cannot negate the some/most inference chain; while we can conclude that there are some Wednesdays without poetry readings, it doesn't follow that those Wednesdays definitely don't have coffee deals. It is possible that regardless of the relationship given, Zack's also offers half-priced coffee every Wednesday.

Without the intervention of conservationists, squirrel monkeys will become extinct. But they will survive if large tracts of second-growth forest habitat are preserved for them. Squirrel monkeys flourish in second-growth forest because of the plentiful supply of their favorite insects and fruit. Which one of the following can be properly inferred from the ecologist's statements? A. No habitat other than second-growth forest contains plentiful supplies of squirrel monkeys' favorite insect and fruit. B. At least some of the conservationists who intervene to help the squirrel monkeys survive will do so by preserving second-growth forest habitat for the monkeys C. Without plentiful supplies of their favorite insects and fruit, squirrel monkeys will become extinct. D. If conservationists intervene to help squirrel monkeys survive, then the squirrel monkey will not become extinct. E. Without the intervention of conservationists, large tracts of second-growth forest habitat will not be preserved for squirrel monkeys.

INFERENCE P1: -intervention --> extinction (contrapositive: -extinction --> intervention) P2: second-growth forest preservation --> -extinction (contrapositive: extinction --> -second-growth forest preservation) Inference: second-growth forest preservation --> -extinction --> intervention OR -intervention --> extinction --> -second-growth forest preservation A. Incorrect. Unsupported. B. Incorrect. We know that second-growth forest preservation --> -extinction --> intervention, but this answer choice assumes intervention --> forest preservation. Furthermore, this answer choice is causal ("will do so by..."). Conditional statements don't guarantee causality. Finally, be wary of "SOME"--remember that conditionals are all or nothing guarantees. C. Incorrect. We know that second-growth forest preservation --> -extinction, but this answer choice assumes -second-growth forest preservation --> extinction. D. Incorrect. We know -intervention --> extinction, but this answer choice assumes intervention --> -extinction. E. Correct. We can infer -intervention --> -second-growth forest preservation from -intervention --> extinction --> -second-growth forest preservation.

Unusually large and intense forest fires swept the tropics in 1997. The tropics were quite susceptible to fire at that time because of the widespread drought caused by an unusually strong El Niño, an occasional global weather phenomenon. Many scientists believe that the strength of the El Niño was enhanced by the global warming caused by air pollution. Which one of the following can be properly inferred from the information above? A. Air pollution was largely responsible for the size and intensity of the forest fires that swept the tropics in 1997. B. If the El Niño in 1997 had not been unusually strong, few if any large and intense forest fires would have swept the tropics in that year. C. Forest fires in the tropics are generally larger and more intense than usual during a strong El Niño. D. At least some scientists believe that air pollution was responsible for the size and intensity of the forest fires that swept the tropics in 1997. E. If air pollution enhanced the strength of the El Niño in 1997, then it also contributed to the widespread drought in that year.

INFERENCE Premises: 1. Air pollution causes global warming 2. Many scientists believe that global warming enhanced the El Nino's unusual strength 3. The unusual strength of the El Nino caused widespread drought 4. The widespread drought made the tropics more susceptible to forest fires. 5. Large and intense forest fires swept the tropics after the El Nino. Analysis: pollution may or may not have made the fires stronger. A. Incorrect. "Largely responsible" is too extreme. Air pollution might have made El Nino stronger. But the drought could still have been strong enough even without the air pollution. B. Incorrect. "Few if any" is too specific/unsupported and extreme. Furthermore, a strong El Niño makes the tropics more susceptible to fire but may or may not influence the size or intensity of fires. C. Incorrect. "Generally" is too broad. Cannot extrapolate beyond one specific instance in 1977. There may be other factors in other years that prevent fires. D. Incorrect. Tempting. We are given that some scientists believe that air pollution, which causes global warming, intensified the El Niño. However, "responsible" is causal and unsupported; cannot identify air pollution as the only cause; other factors (ex: the drought itself was strong enough without air pollution) may have instead caused the size and intensity of the forest fires. E. Correct. We know that the El Nino caused widespread drought. If we accept the scientists' belief ("the strength of the El Nino was enhanced by the global warming caused by air pollution") then the air pollution also contributed to the drought.

In modern "brushless" car washes, cloth strips called mitters have replaced brushes. Mitters are easier on most cars' finishes than brushes are. This is especially important with the new clear-coat finishes found on many cars today, which are more easily scratched than old finishes are. Which one of the following is most strongly supported by the statements above, if those statements are true. A. When car washes all used brushers rather than mitters, there were more cars on the road with scratched finishes than there are today. C. Modern "brushless" car washes usually do not produce visible scratches on wars with older finishes.

INFERENCE Remember that "most strongly supported" does not mean "can be inferred." You don't need to prove anything on this sort of question. You just need to find an answer choice that enjoys more support than the other answer choices. A. Incorrect. Too extreme. We cannot infer that brushes caused scratches; we only know that mitters are easier on cars than brushers. Older finishes were less easily scratched than the new clear-coat finishes. Maybe they held up easily to the brushes before. C. Correct. Since mitters are better for clear-coat finishes, they should be even better for older finishes. While we cannot prove "usually," there is strong support that mitters will be much easier on older finishes than newer ones - leading us to suspect that they usually don't leave scratches on older finishes. This one is all about the question stem!

People who object to the proposed hazardous waste storage site by appealing to extremely implausible scenarios in which the site fails to contain the waste safely are overlooking the significant risks associated with delays in moving the waste from its present unsafe location. If we wait to remove the waste until we find a site certain to contain it safely, the waste will remain in its current location for many years, since it is currently impossible to guarantee that any site can meet that criterion. Yet keeping the waste at the current location for that long clearly poses unacceptable risks. The statements above, if true, most strongly support which one of the following? A. The waste should never have been stored in its current location. B. The waste should be placed in the most secure location that can ever be found. C. Moving the waste to the proposed site would reduce the threat posed by the waste. D. Whenever waste must be moved, one should limit the amount of tie allotted to locating alternative waste storage sites. E. Any site to which the waste could be moved will be safer than its present site.

INFERENCE Scenarios of the proposed storage site failing are implausible Unacceptable risk in delays in moving waste from unsafe site. Wait to find safe site --> waste will remain in unsafe site. A. Incorrect. No information about what happen previously. B. Incorrect. "Most secure" is too extreme. C. Correct. Keeping the waste at the current location unacceptable risks, and objections to the proposed site are implausible, so we can infer that moving waste out of the current location and moving it to the proposed site would be better. D. Incorrect. "Whenever" is too extreme. The normative "should" and the discussion of "time allotted" are unsupported. E. Incorrect. "Any" is too extreme.

Decentralization enables divisions of a large institution to function autonomously. This always permits more realistic planning and strongly encourages innovation, since the people responsible for decision making are directly involved in implementing the policies they design. Decentralization also permits the central administration to focus on institution-wide issues without being overwhelmed by the details of daily operations. The statements above most strongly support which one of the following? A. In large institutions whose divisions do not function autonomously, planning is not maximally realistic C. For large institutions, the advantages of decentralization outweigh its disadvantages D. The central administrations of large institutions are usually partially responsible for most of the details of daily operations.

INFERENCE 1. Decentralization --> autonomy 2. Autonomy --> responsible for implementing own policies --> innovation 3. Decentralization --> focus on institution-wide issues A. Correct. C. Incorrect. Not enough information on disadvantages to make this claim. D. Incorrect. "Most of the details of daily operations" is unsupported. Furthermore, this answer choice assumes that most large institutions are CENTRALIZED, ingoring the fact that there may be more large institutions that are decentralized and therefore NOT usually partially responsible for most of the details of daily operations.

Most opera singers who add demanding roles to their repertoires at a young age lose their voices early. It has been said that this is because their voices have not yet matured and hence lack the power for such roles. But young singers with great vocal power are the most likely to ruin their voices. The real problem is that most young singers lack the technical training necessary to avoid straining their vocal cords--especially when using their full vocal strength. Such misuse of the cords inevitably leads to a truncated singing career. Which one of the following does the information above most strongly support? A. Young opera singers without vocal powers are unlikely to ruin their voices by singing demanding roles. B. Some young opera singers ruin their voices while singing demanding roles because their vocal cords have not yet matured. C. Only opera singers with many years of technical training should try to sing demanding roles. D. Only mature opera singers can sing demanding roles without undue strain on their vocal cords. E. Most young opera singers who sing demanding roles strain their vocal cords.

INFERENCE A. Incorrect. "Unlikely" is unsupported. B. Incorrect. Author disagrees with this inference. C. Incorrect. Unsupported. D. Incorrect. "Only" is out of scope. E. Correct. Inferred from the facts... 1. Most young opera singers with demanding roles lose their voices 2. Most young singers lack the technical training necessary to avoid straining their vocal cords

Most large nurseries sell raspberry plants primarily to commercial raspberry growers and sell only plants that are guaranteed to be disease-free. However, the shipment of raspberry plants that Johnson received from Wally's Plants carried a virus that commonly afflicts raspberries. Which one of the following is most strongly supported by the information above? A. If Johnson is a commercial raspberry grower and Wally's Plants is not a large nursery, then the shipment of raspberry plants that Johnson received was probably guaranteed to be disease-free. B. Johnson is probably not a commercial raspberry grower if the shipment of raspberry plants that Johnson received from Wally's Plants was not entirely as it was guaranteed to be. C. If Johnson is not a commercial raspberry grower, then Wally's Plants is probably not a large nursery. D. Wally's Plants is probably not a large, well-run nursery if it sells its raspberry plants primarily to commercial raspberry growers. E. If Wally's Plants is a large nursery, then the raspberry plants that John received in the shipment were probably not entirely as they were guarantee to be.

Inference Large nurseries (most) --> sell to commercial growers. Large nurseries (most) --> sell with disease-free guarantee. It doesn't matter if Johnson is commercial. It doesn't matter if the plants were guaranteed. Those are "necessary," not "sufficient" conditions of the most statement. Knowing those doesn't tell us anything, because you can't read "most" statement backwards or take contrapositives. A. Incorrect. Commercial growers are on the left side; they are not sufficient. Furthermore, we can not make inferences about not large nurseries. B. Incorrect. Disease-guarantees are on the left side; they are not sufficient. Furthermore, most + most = some. We can only conclude that some nurseries that sell to commercial raspberry growers also sell only plants guaranteed to be disease-free. Some could range from 1 to all. Thus, if the shipment was guaranteed to be disease-free, we cannot predict the probability that the nursery sold to a commercial raspberry grower (or in other words that Johnson is a commercial raspberry grower). C. Incorrect. We do not have information about non-commercial growers. Furthermore, commercial growers are on the left side; they are not sufficient. It is given that some (50% or less) large nurseries do not sell to commercial raspberry growers, so John not being a commercial raspberry grower is acceptable. Remember that you cannot take the contrapositive of the most statement (which is what this answer choice does!) D. Incorrect. "Well-run" is out of scope. Furthermore, commercial growers are on the left side; they are not sufficient. It is given that most (greater than 50%) large nurseries do sell to commercial raspberry growers. Finally, we can never conclude that Wally's is probably not a large nursery, since being a large nursery is on the sufficient side. E. Correct. If Wally's Plants is a large nursery, we know that at least 51% of the time it sells only plants that are guaranteed to be disease-free. In other words, given that Wally's Plants is a large nursery and that Johnson received plants with a virus, then it is more likely (51+% chance) that Wally's Plants weren't as guaranteed (the plants were diseased. Therefore, if there was a guarantee, then the plants weren't as guaranteed).

In the Riverview Building, every apartment that has a balcony also has a fireplace. None of the apartments with balconies is a one-bedroom apartment. So none of the one-bedroom apartments has a fireplace. The flawed nature of the argument above can most effectively be demonstrated by noting that, by parallel reasoning, we could conclude that A. Every fish has fur since no cat lacks fur and no cat is a fish. B. Some cats lack fur since every dog has fur and no cat is a dog. C. No dog has fur since every cat has fur and no cat is a dog. D. Every cat is a fish since no cat is a dog and no dog is a fish. E. No fish is a dog since every dog is a mammal and no fish is a mammal.

MATCH THE FLAW B —> F. B —> -one bed. One bed —> -F. A. Incorrect. Should conclude fish —> -fur. Cat —> fur. Cat —> -fish. Fish — > fur. B. Incorrect. "Some" does not match. C. Correct. Matches flaw. Cat —> fur. Cat —> -dog. Dog —> -fur. D. Incorrect. Cat —> -dog. Dog —> -fish. Cat —> fish. E. Incorrect. Dog —> mammal. Fish —> -mammal. Fish —> -dog

Every brick house on River Street has a front yard. Most of the houses on River Street that have front yards also have two stories. So most of the brick houses on River Street have two stories. Which one of the following is most appropriate as an analogy demonstrating that the reasoning in the argument above is flawed? A. By that line of reasoning, we could conclude that most politicians have run for office, since all legislators are politicians and most legislators have run for office. B. By that line of reasoning, we could conclude that most public servants are legislators, since most legislators have run for office and most politicians who have run for office are public servants. C. By that line of reasoning, we could conclude that not every public servant has run for office, since every legislator is a public servant but some public servants are not legislators. D. By that line of reasoning, we could conclude that most legislators have never run for office, since most public servants have never run for office and all legislators are public servants. E. By that line of reasoning, we could conclude that most legislators are not public servants, since most public servants have not run for office and most legislators have run for office.

MATCH THE FLAW BH --> front yard. Front yard -(most)-> 2 stories. BH -(most)-> 2 stories. A. Incorrect. L --> P L -(most)-> office. P -(most)-> office. B. Incorrect. P and office -(most)-> PS L -(most)-> office. PS -(most)-> L. C. Incorrect. L --> PS. PS -(some)-> -L. PS -(not all)-> office. D. Correct. L --> PS. PS -(most)-> -office. L -(most)-> -offce. E. Incorrect. L -(most)-> office. PS -(most)-> -office. L -(most)-> -PS.

Advertisement: Our oat bran cereal is the only one that has printed right on its package all of its claimed health benefits. And really health-conscious consumers have demonstrated that these health claims are true by buying our cereal since they would not have bought our cereal unless the claims were true. How do we know these consumers are really health-conscious? No really-health conscious consumer would buy food in a package that did not have accurate information about the food's health benefits printed on it. Which one of the following employs a flawed argumentative strategy that is most closely parallel to the flawed argumentative strategy in the advertisement above? B. This card game must be intellectually challenging because it is played by highly intelligent people, who play only intellectually challenging card games. In fact, these players' intelligence is demonstrated by the fact that they play this game. E. This painting is a poor imitation because only people with poor eyesight mistook it for the original. That these people have poor eyesight is demonstrated by the fact that they also mistook a vase of blowers in the painting for a peacock.

MATCH THE FLAW P1: -healthy --> -health-conscious consumers purchase cereal P2: In fact, we know that these people are healthy because they buy foods with health claims. IC: health-conscious consumers purchase cereal C: health claims are true Circular reasoning B. Correct. Circle reasoning E. Incorrect. The final sentence does not mirror circular reasoning but rather offers support for people having poor eyesight.

Each of the smallest particles in the universe has an elegantly simple structure. Since these particles compose the universe, we can conclude that the universe itself has an elegantly simple structure. Each of the following arguments exhibits flawed reasoning similar to that in the argument above EXCEPT: A. Each part of this car is nearly perfectly engineered. Therefore, this car is nearly perfect, from an engineering point of view. B. Each part of this desk is made of metal. Therefore, this desk is made of metal. C. Each brick in this wall is rectangular. Therefore, this wall is rectangular. D. Each piece of wood in this chair is sturdy. Therefore, this chair is sturdy. E. Each sentence in this novel is well-constructed. Therefore this is a well-constructed novel.

MATCH THE FLAW Part v. whole flaw A. Incorrect. Matches flaw. B. Correct. Valid argument; not flawed. C-E. Incorrect. Match flaw.

Decreased reliance on fossil fuels is required if global warming is to be halted. The current reliance would decrease if economic incentives to develop alternative energy sources were present. So ending global warming requires offering economic incentives to develop alternative energy sources. The flawed pattern of reasoning exhibited by the argument above most closely parallels that exhibited by which one of the following? A. If we end poverty we will end hunger. Ending unemployment will end poverty, So ending unemployment will end hunger. D. Keeping good teachers is necessary for improving education. If teachers' salaries were improved, good teachers would remain in the profession. So an increase in teachers' salaries is necessary to improve education.

MATCH THE FLAW P1: -GW --> -reliance P2: Economic incentives --> -reliance C: -GW --> economic incentives A. Incorrect; logically valid -Poverty --> -hunger -Unemployment --> -poverty -Unemployment --> -hunger D. Correct Improve edu --> good teachers Improve salaries --> good teachers Improve edu --> improve salaries

Each new car in the lot at Rollway Motors costs more than $18,000. Any car in their lot that is ten or more years old costs less than $5,000. Thus, if a car in Rollway's lot costs between $5,000 and $18,000, it is a used car that is less than 10 years old. The pattern of reasoning in which one of the following arguments is most similar to that in the argument above? A. Each apartment above the fourth floor of the building has more than two bedrooms. But all apartments below the fourth floor have fewer than two bedrooms. Thus, any apartment on the fourth floor of the building has exactly two bedrooms. B. Each apartment above the fourth floor has 2 or 3 bedrooms. But no apartment below the fourth floor has more than 2 bedrooms. Thus, all of the building's 3 bedroom apartments are on the fourth floor or higher. C. No apartment above the fourth floor of the building has fewer than three bedrooms. But all apartments below the fourth floor have fewer than two bedrooms. Thus, if there are apartments in the building with exactly two bedrooms, they are on the fourth floor. D. No apartment above the fourth floor of the building has more than two bedrooms. But only 3-bedroom apartments have balconies. Thus, if any apartment in the building has a balcony, it is on the fourth floor or lower. E. Each apartment above the fourth floor of the building has more than two bedrooms. The building has no vacant apartments on or below the fourth floor. Thus, if there is any vacant apartment in the building, it will have more than two bedrooms.

MATCH THE REASONING New --> 18,000 >10 years --> <5,000 between 5,000 and 18000 --> <10 years old The conclusion should invert the relationship between the two elements (X and Y) in the premises and adopt a range within Y to predict -X A. Incorrect. Does not invert the relationship between two initial elements. Above 4th --> >2 bedrooms Below 4th --> <2 bedrooms Between two floors/4th floor --> 2 bedrooms B. Incorrect. Does not correctly identify range within Y (should be between 2/3 and 2 or fewer). Above 4th --> 2 or 3 Below 4th --> no >2 bedrooms/2 or fewer bedrooms 3 bedrooms --> 4th floor or higher C. Correct Above 4th --> no <3/3 or more bedrooms Below 4th --> <2 bedrooms 2 bedrooms --> 4th floor D. Incorrect. Does not adopt range structure Above 4th --> no >2 bedrooms/2 or fewer bedrooms 3-bedroom apartments --> balconies Balcony --> 4th floor or lower E. Incorrect. Confuses elements Above 4th --> >2 bedrooms On or below 4th --> no vacant apartments Vacant apartment --> 2 bedrooms

Editorial: In order to encourage personal responsibility in adults, society should not restrict the performance of any of the actions of adults or interfere with the likely results except to prevent negative effects on others. Which one of the following expresses a view that is inconsistent with the principle stated in the editorial? C. Even though public smoking may lead to indirect harm to others, it should not be banned. There are several other ways to eliminate this harm that do not restrict the conduct of smokers and hence are preferable to a complete ban on public smoking. E. It is not enough that consumable products containing harmful substances have warning labels. Many adults simply ignore such warnings and continue to consume these substances in spite of the harm it may cause them. This is why consuming such substances should be illegal.

MUST BE FALSE Principle: If not to prevent negative effects to others --> do not restrict. If restrict --> to prevent negative effect to others. C. Incorrect. "There are several other ways to eliminate this harm" satisfies "not to prevent negative effects to others." This justifies "do not restrict. E. Correct. The harm described is to ONESELF, not to ANOTHER. Fails to comply with the principle/justify the restriction.

Chef: This mussel recipe's first step is to sprinkle the live mussels with cornmeal. The cornmeal is used to clean them out: they take the cornmeal in and eject the sand they contain. But I can skip this step, because the mussels available at seafood markets are farm raised and therefore do not contain sand. Which one of the following is an assumption required by the chef's argument? A. Cornmeal is not used to clean out farm-raised mussels before they reach seafood markets. B. Mussels contain no contaminants other than sand. C. Sprinkling the mussels with cornmeal does not affect their taste. D. The chef's mussel recipe was written before farm-raised mussels became available. E. The mussels the chef is using for the mussel recipe came from a seafood market.

NECESSARY ASSUMPTION A. Incorrect. Not necessary--already given that the mussels do not contain sand. B. Incorrect. We are concerned about cleaning out sand, not other contaminants in the first step. C. Incorrect. We are concerned about cleaning out sand, not taste in the first step. D. Incorrect. Would weaken the argument that the chef can skip this step. E. Correct. Fails the negation test. If we negate this and the chef's mussels aren't from the seafood market, they might have sand and thus require the first step.

Film preservation requires transferring old movies from their original material--unstable, deteriorating nitrate film--to stable acetate film. But this is a time-consuming, expensive process, and there is no way to transfer all currently deteriorating nitrate films to acetate before they disintegrate. So some films from the earliest years of Hollywood will not be preserved. Which one of the following is an assumption on which the argument depends? A. No new technology for transferring old movies from nitrate film to acetate film will ever be developed. C. Not many films from the earliest years of Hollywood have already been transferred to acetate. D. Some films from the earliest years of Hollywood currently exist in their original material.

NECESSARY ASSUMPTION A. Incorrect. Tempting, but we cannot assume that no new technology will ever be developed. We only have to assume that no improved technology will be developed before the old films deteriorate. C. Incorrect. Fails the negation test: "many films from the earliest years of Hollywood have already been transferred to acetate"; argument still holds since it concludes that only SOME films from the earliest years of Hollywood will not be preserved. D. Correct. Passes the negation test: "no films from the earliest years of Hollywood currently exist in their original material"; argument falls apart, since these films cannot be preserved.

Psychologist: Birth-order effects, the alleged effects of when one was born relative to the births of siblings, have not been detected in studies of adult personality that use standard personality tests. However, they have been detected in birth-order studies that are based on parents' and siblings' reports of the subjects' personalities. All of these birth-order studies, taken together, show that birth order has no lasting effect on personality; instead, birth order affects merely how a sibling's behavior is perceived. Which one of the following is an assumption required by the psychologist's argument? A. Standard personality tests will detect at least some birth-order effects on personality, if those effects exist. C. Parents' and siblings' perceptions of a person's personality tend not to change between that person's early childhood and adulthood.

NECESSARY ASSUMPTION P1: no effects on adult personality via standard tests P2: effects on parents' and siblings' reports of the subjects' personalities C: no lasting effect on personality but on how behavior is perceived. A: standard personality tests offer an accurate view of a person's personality, while reports by family do not A. Correct. Passes negation test: Standard personality tests will never detect some birth-order effects; argument falls apart. C. Incorrect. Fails the negation test. Argument does not assume that these family perceptions are stable or accurate.

Barnes: the two newest employees at this company have salaries that are too high for the simple tasks normally assigned to new employees and duties that are too complex for inexperienced workers. Hence, the salaries and the complexity of the duties of these two newest employees should be reduced. Which one of the following is an assumption on which Barnes's argument depends? A. The duties of the two newest employees are not less complex than any others in the company. B. It is because of the complex duties assigned that the two newest employees are being paid more than is usually paid to newly hired employees. C. The two newest employees are not experienced at their occupations. E. The salaries of the two newest employees are no higher than the salaries that other companies pay for workers with a similar level of experience

NECESSARY ASSUMPTION P1: salaries that are too high, higher than what we pay new employees for the simple tasks we normally assign to them P2: duties that are too complex for inexperienced workers C: we should reduce their salaries and simplify their tasks Assumes that workers are either inexperienced or completing simple tasks A. Incorrect. Not necessary; there could be employees with duties less complex B. Incorrect. Not necessary. C. Correct. "The two newest employees are experienced at their occupations"; the claim that "duties that are too complex for inexperienced workers" falls apart. E. Incorrect. "Other companies" is out of scope

It is popularly believed that a poem has whatever meaning is assigned to it by the reader. But objective evaluation of poetry is possible only if this popular belief is false; for the aesthetic value of a poem cannot be discussed unless it is possible for at least two readers to agree on the correct interpretation of the poem. Which one of the following is an assumption required by the argument? A. Only if they find the same meaning in a poem can two people judge that it has aesthetic value. B. If two readers agree about the meaning of a given poem, that ensures that an objective evaluation of the poem can be made. C. Discussion of a poem is possible only if it is false that a poem has whatever meaning is assigned to it by the reader. D. A given poem can be objectively evaluated only if the poem's aesthetic value can be discussed. E. Aesthetic evaluation of literature is best accomplished through discussion by more than two readers.

NECESSARY ASSUMPTION P: -Two readers agree --> -discuss aesthetic value Discuss aesthetic value --> two readers agree C: Objective evaluation --> -reader assigns meaning. A: The assumption must link discussing aesthetic value to objective evaluation and/or -reader assigns meaning to two readers agree. The argument assumes that discussing aesthetic value is necessary for objective evaluation; objective evaluation depends on the discussion of aesthetic value and thus the reader-interpretation belief being false! The assumption is that those two ideas are somehow related. Or in other words, aesthetic value is relevant to objective evaluation. And go through the answer choices and find out for yourself that only (D) makes any sort of connection between those two ideas. A. Incorrect. Premise booster. Furthermore, "judge aesthetic value" differs from "discuss aesthetic value." B. Incorrect. Illegal negation; two readers agree --> objective evaluation, yet two readers agree is never the sufficient condition. Instead, the author believes -two readers agree --> -objective evaluation. C. Incorrect. Discussion --> -reader assigns meaning. However, "discussion" it too broad; the stimulus only specifies "discuss aesthetic value." D. Correct. Objective evaluation --> discuss aesthetic value. E. Incorrect. "Best" is out of scope.

The ability of mammals to control their internal body temperatures is a factor in the development of their brains and intelligence. This can be seen from the following facts: the brain is a chemical machine, all chemical reactions are temperature dependent, and any organism that can control its body temperature can assure that these reactions occur at the proper temperatures. Which one of the following is an assumption on which the argument depends? A. Organisms unable to control their body temperatures do not have the capacity to generate internal body heat without relying on external factors. B. Mammals are the only animals that have the ability to control their internal body temperatures. C. The brain cannot support intelligence if the chemical reactions within it are subject to uncontrolled temperatures. D. The development of intelligence in mammals is not independent of the chemical reactions in their brains taking place at the proper temperature. E. Organisms incapable of controlling their internal body temperatures are subject to unpredictable chemical processes.

NECESSARY ASSUMPTION P: Brain --> chemical machine --> proper temperature. Control internal temperature --> proper temperature. C: Controlling internal temperature is a factor influencing the development of brains and intelligence. A: connect evidence about brains to evidence about brains and intelligence. A. Incorrect. Negation: Organisms unable to control their body temperatures do not have the capacity to generate internal body heat without relying on external factors; argument holds; other causes for proper temperature do not weaken the premise control internal temperature --> proper temperature. B. Incorrect. Not necessary. C. Incorrect. Conclusion repeater that is too extreme. While the answer choice states that controlling chemical reactions is A factor to brain development, it never concludes or assumes that it's an essential element of developing intelligence. D. Correct. Negation: the development of intelligence in mammals IS independent of the chemical reactions in their brains taking place at the proper temperature; argument falls apart, as P is specific to the brain but C describes both the brain and intelligence. E. Incorrect. Premise booster. -Proper temperature --> -chemical machine.

There are far fewer independent bookstores than there were 20 years ago, largely because chain bookstores prospered and multiplied during that time. Thus, chain bookstores' success has been to the detriment of book consumers, for the shortage of independent bookstores has prevented the variety of readily available books from growing as much as it otherwise would have. Which one of the following is an assumption on which the argument relies? A. Book consumers would be better off if there were a greater variety of readily available books than there currently is. B. Independent bookstores typically do not sell the kinds of books that are available in chain bookstores.

NECESSARY ASSUMPTION P: Chain bookstores' success --> independent bookstores' decline. Independent bookstores' decline --> shortage of readily available books. C: Chain bookstores' success harms book consumers A: Shortage of readily available books harms book consumers. A. Correct. Passes the negation test. B. Incorrect. Not necessary

Sometimes one reads a poem and believes that the poem expresses contradictory ideas, even if it is a great poem. So it is wrong to think that the meaning of a poem is whatever the author intends to communicate to the reader by means of the poem. No one who is writing a great poem intends it to communicate contradictory ideas. Which one of the following is an assumption on which the argument depends? A. Different readers will usually disagree about what the author of a particular poem intends to communicate by means of that poem. C. Readers will not agree about the meaning of a poem if they do not agree about what the author of the poem intended the poem to mean. E. If a reader believes that a poem expresses a particular idea, then that is the meaning of the poem.

NECESSARY ASSUMPTION P: Readers sometimes think poems express contradictory ideas. Poets do not intend to communicate contradictory ideas. C: Meaning of the poem does not = what the poet intends to communicate (in other words, based on the assumption, what the reader thinks is the poem's ideas does not = what the poet intends to communicate). A: What the reader thinks is the poem's ideas = the meaning of the poem. A. Incorrect. We already know readers may have different ideas. Not necessary. Possible that all readers may disagree yet still think that the poem expresses ideas that do not = what the author intends to communicate (allowing the argument to still hold). C. Incorrect. We already know readers may have different ideas. Furthermore, this challenges the idea that the meaning of a poem = what the author of the poem intended to communicate. The argument does not depend on this assumption but rather on the assumption that that the meaning of a poem = what the reader thinks is the poem's ideas. E. Correct. Exact statement of assumption.

Corporate businesses, like species, must adapt to survive. Businesses that are no longer efficient will become extinct. But sometimes a business cannot adapt without changing its core corporate philosophy. Hence, sometimes a business can survive only by becoming a different corporation. Which one of the following is an assumption required by the argument? A. No business can survive without changing its core corporate philosophy. B. As a business becomes less efficient, it invariably surrenders its core corporate philosophy. C. Different corporations have different core corporate philosophies. D. If a business keeps its core corporate philosophy intact, it will continue to exist. E. A business cannot change its core corporate philosophy without becoming a different corporation.

NECESSARY ASSUMPTION P: Survive --> adapt -Efficient --> extinct --> -survive -Change core philosophy --> -adapt C: Survive -(some)-> different corporation A: Assumes change core philosophy --> different corporation in order to allow survive -(some)-> adapt --> change core philosophy --> different corporation A. Incorrect. Assumes that businesses survive only when changing their core corporate philosophies. C only states that SOME businesses can survive only by changing core philosophy and thus becoming a different corporation. B. Incorrect. Does not deal with gap between changing core philosophy and becoming a different corporation. "Less efficient" is out of scope. C. Incorrect. Illegal reversal of the desired assumption; different corporation --> change core philosophy. However, would be correct if it read "at least some different corporations have different core philosophies." D. Incorrect. Change core philosophy --> survive. Does not deal with gap between changing core philosophy and becoming a different corporation. E. Correct.

Large-scale government projects designed to benefit everyone--such as roads, schools, and bridges--usually benefit some small segments of society, initially at least, more than others. The more equally and widely political power is distributed among the citizenry, the less likely such projects are to receive funding. Hence, government by referendum rather than by means of elected representatives tends to diminish, not enhance, the welfare of society. Which one of the following is an assumption on which the argument depends? A. Large scale government projects sometimes enhance the welfare of society. E. Government by referendum is the only way to distribute political power equally and widely

NECESSARY ASSUMPTION P: Equal power --> -large-scale government projects C: Government by referendum --> -welfare A: Referendum --> equal power --> -welfare --> -large-scale government projects A. Correct. Negation: large-scale government projects never enhance the welfare of society; argument falls apart E. Incorrect. Negation: government by referendum is not the only way to distribute political power equally and widely; argument does not fall apart. "Only" is too extreme. Referendum must be a way but need not be the ONLY way to distribute political power equally and widely.

Art history: this painting, purportedly by Mary Cassatt, is a forgery. Although the canvas and other materials are consistent with most of Cassatt's work, and the subject matter is similar to that of Cassatt's finest paintings, the brush style of this painting is not found in any work known to be Cassatt's. Hence this painting is definitely not the work of the genuine Cassatt's. The art historian's argument depends on assuming which of the following? A. The type of canvas and other materials that Cassatt used in most of her work were readily available to others. B. None of Cassatt's works is painted using a brush style that is not exhibited in any of her known works. C. Cassatt's work generally had a characteristic subject matter that distinguished it from the work of other painters of her era. D. The most characteristic feature of Cassatt's work is her brush style. E. No painter other than Cassatt would be able to match Cassatt's style perfectly.

NECESSARY ASSUMPTION P: brush style of painting is not consistent with any of Cassatt's known work C: painting is not by Cassatt A. Incorrect. "Canvas and other materials" are out of scope. B. Correct. In other words, all of Cassatt's known work use the same brush style. C. Incorrect. Subject matter is out of scope. D. Incorrect. Not necessary. Fails the negation test: the most characteristic feature of Cassatt's work is not her brush stroke. E. Incorrect. Does not address fact that the painting in question lacks Cassatt's brush stroke.

Historian: Radio drama requires its listeners to think about what they hear, picturing for themselves such dramatic elements as characters' physical appearances and spatial relationships. Hence, while earlier generations, for whom radio drama was the dominant form of popular entertainment, regularly exercised their imaginations, today's generation of television viewers do so less frequently. Which one of the following is an assumption required by the historian's argument? A. People spend as much time watching television today as people spent listening to radio in radio's heydey. D. For today's generation of television viewers, nothing fills the gap left by radio as a medium for exercising the imagination. E. Television drama does not require its viewers to think about what they see.

NECESSARY ASSUMPTION P: radio requires listeners to picture what they hear. C: TV viewers do not exercise imagination as much as radio listeners do. A: The argument links picturing elements to creativity. The argument assumes that some television lacks the imaginative quality of radio. A. Incorrect. Even if the time spent is different, television viewers may still exercise their imagination less frequently. D. Correct. E. Incorrect. This need not be true. Too weak to address the assumption that television lacks imagination. Even though TV does not require viewers to think, they may still exercise imagination in other way. Furthermore, "television drama" is too specific.

Fund-raiser: A charitable organization rarely gives its donors the right to vote on its policies. The inability to directly influence how charities spend contributions makes potential donors feel less of an emotional connection to the charity. Thus, most charities could probably increase the amount of money they raise through donations by giving donors the right to vote. Which one of the following is an assumption that the fundraiser's argument depends on? A. The most effective way for a charity to give potential donors the ability to directly influence what the charity does is by giving donors the right to vote on the charity's policies. B. Most charities that have increased the amount of money they raise through donations have done so by making potential donors feel a greater emotional connection to the charity. C. Every charity that has given donors the right to vote on its policies has seen a marked increase in the emotional connection donors have to that charity. D. Most potential donors to a charity are unwilling to give that charity any money if there is no possible way for them to have any influence of that charity's policies. E. The emotional connection potential donors feel to a charity can affect the amount of money that charity raises through donations.

NECESSARY ASSUMPTION Premise: No voting --> no influence --> less emotional connection C: Voting --> probably increase $ A: Voting --> influence --> more emotional connection --> probably increase $ A. Incorrect. out of scope. The argument is not interested in the most effective way to give donors control of the organization but rather the most effective way to get more donations. B. Incorrect. Supports the argument but fails the negation test: less than half of charities that have increased the amount of money they raise through donations have done so by making potential donors feel a greater emotional connection to the charity. Argument does not fall apart. C. Incorrect. Premise booster that does not fill in the gap; links voting --> more emotional connection, but we need we really to link emotional connection --> probably increase $. D. Incorrect. Premise booster that does not fill in the gap; links -influence --> -probably increase $, but we need we really to link emotional connection --> probably increase $. E. Correct. Passes the negation test: "The emotional connection donors feel CANNOT AFFECT the amount of money raised through donations"; argument falls apart

If there are sentient beings on planets outside our solar system, we will not be able to determine this anytime in the near future unless some of these beings are at least as intelligent as humans. We will not be able to send spacecrafts to planets outside our solar system anytime in the near future, and any sentient being on another planet capable of communicating with us anytime in the near future would have to be at least intelligent as we are. The argument's conclusion can be properly inferred if which one of the following is assumed? A. There are no sentient beings on planets in our solar system other than Earth. B. Any beings that are at least as intelligent as humans would want to communicate with sentient beings outside their own solar systems. C. If there is a sentient being on another plant that is as intelligent as humans are, we will not be able to send a spacecraft to the being's planet anytime in the near future. D. If a sentient being on another planet cannot communicate with us, then the only way detect its existence is by sending a spacecraft to its planet. E. Any sentient being on planets outside our solar system that are at least as intelligent as humans would be capable of communicating with us.

P: -send spacecrafts sentient beings communicate with us --> sentient beings as intelligent as humans C: -sentient beings as intelligent as humans --> -determine existence; this contrapositive is determine existence --> sentient beings as intelligent as humans A involving the P2: determine existence --> sentient beings communicate with us OR A involving P1: -send spacecrafts --> -determine existence; this contrapositive is determine existence --> send spacecrafts Thus... A: determine existence --> sentient beings communicate with us OR end spacecrafts A. Incorrect. Sentient beings inside our solar system besides humans are out of scope. B. Incorrect. "Want" is out of scope C. Incorrect. Sentient beings as intelligent as humans --> -send spacecrafts. Does not connect either premise with the ability to determine existence. D. Correct. -Sentient beings communicate with us. Determine existence --> send spacecraft E. Incorrect. Reversal of P2: sentient beings as intelligent as humans --> sentient beings communicate with us

Rachel: Through contemporary artists are pleased to be free of the constraints that bound their predecessors, this freedom has caused a decline in the quality of art. Great art can be produced only when artists struggle to express themselves within externally imposed boundaries. James: People have always been critical of the art of their own time. They forget all but the greatest art from past eras. Since inferior contemporary artworks have not yet been forgotten, people today mistakenly think that contemporary art is generally inferior to earlier art. On the basis of their statements, Rachel and James are committed to disagreeing with each other about whether A. Contemporary art is of lower quality than earlier art. D. Inferior art from past eras is generally forgotten.

POINT OF DISAGREEMENT A. Correct. Rachel believes that contemporary art is of lower quality than earlier art, while James believes that people are mistaken to believe that contemporary art is of lower quality than earlier art. D. Incorrect. James disagrees. Rachel's opinion is unknown; she believes that contemporary art is not constrained by the boundaries of past art, but this does not mean that past art has been forgotten.

Jenkins: Reason on the properties of snow at the North Pole should be conducted in January and February. The weather is then cold enough to ensure that the snow will not melt. It is important that research money not be wasted; if we wait until a later month, we risk sending researchers when they will be unable to carry out research successfully. Lurano: I disagree. The weather will likely still be quite cold in April and May, and by going later, researchers run less risk of suffering dangerous exposure to the cold. The dialogue lends the most support to the claim that Jenkins and Lurano disagree on whether... A. There is a possibility of snow melting in the North Pole during April and May. B. It is impossible to investigate the properties of snow at the North Pole later than February. C. Funding will be wasted if research on snow is carried out at the North Pole later than February. D. The temperatures at the North Pole in January and February are lower than are temperatures in April and May. E. Research funding considerations outweigh the risk to researchers posed by the temperatures at the North Pole in January and February.

POINT OF DISAGREEMENT A. Incorrect. Both agree B. Incorrect. Too extreme, neither agree C. Incorrect. Jenkins agrees, but Lurano makes no statement about funding. D. Incorrect. Both agree E. Correct. Stated by Lurano, implied by Jenkins. Jenkins states that funding is the primary concern and thus that it outweighs the risk to researchers.

Real estate agent: upon selling a hoe, the sellers are legally entitled to remove any items that are not permanent fixtures. Legally, large appliances like dishwashers are not permanent fixtures. However, since many prospective buyers of the home are likely to assume that large appliances in the home would be included with its purchase, sellers who will be keeping the appliances are morally obligated either to remove them before showing the home or indicate in some way that the appliances are not included. Which one of the following principles, if valid, most helps to justify the real estate agent's argument? A. If a home's sellers will be keeping any belongings that prospective buyers of the home might assume would be included with the purchase of the home, the sellers are morally obligated to indicate clearly that those belongings are not included. B. A home's sellers are morally obligated to ensure that prospective buyers of the home do not assume that any large appliances are permanent fixtures in the home. D. A home's sellers are morally obligated not to deliberately mislead any prospective buyers of their home about which belongings are included with the sale of the home or not.

PRINCIPAL SUPPORT Legally required to remove not permanent fixtures. Buyers assume that large appliances would be included. If keeping appliances --> morally obligated to remove OR indicate that they are not included. A. Correct. Principle uses stronger language of "any belongings" to capture the appliances described in the stimulus. A is not perfect but rather the most justified. B. Incorrect. The argument is not concerned that buyers assuming that large appliances are permanent fixtures but rather that they are included in purchase. Furthermore, the stimulus only tells us that people are morally obliged 1. remove the appliances before showing the home OR 2. indicate in some other way that the appliances are not included. Nowhere in the stimulus does it say that a seller is morally obliged TO "ensure that prospective buyers of the home do not assume that any large appliances are permanent fixtures in the home." Finally, the conclusion is limited to people who will be KEEPING some appliances. B doesn't have this limitation. So if you chose B, you'd have to tell them they aren't permanent fixtures - even if you planned to give them away. D. Incorrect. The argument never suggests that homeowners are intentionally misleading anybody.

A first-term board member should not be on the finance committee unless he or she is an accountant or his or her membership on the committee is supported by all members of the board. Which one of the following arguments most closely conforms to the principle stated above? A. Simkins is a first-term board member and not an accountant; thus, Simkins should not be on the finance committee. D. Klein is a first-term board member who is not an accountant; thus, Klein should not be allowed on the finance committee if any board member opposes Klein's appointment to that committee.

PRINCIPLE EXAMPLE -(Accountant OR unanimous board support) → -finance committee -Accountant AND -unanimous board support → -finance committee Finance committee → accountant OR unanimous board support DO NOT forget to distribute the negative: -accountant OR -unanimous board support → -finance committee is incorrect A. Incorrect. Conforms to the incorrect diagram. Lacks -unanimous board support. D. Correct. -Accountant. If -supported --> -finance committee.

Doctor: It is wrong for medical researchers to keep their research confidential, even if the companies for which they work would rather that they do so. If research results are not shared, the development of effective medical treatments may be delayed, and thus humans may suffer unnecessarily. Which one of the following principles, if valid, most helps to justify the doctor's argument? A. Medical researchers should never engage in any behavior that they know will cause humans to suffer. B. If the most important moral principle is to prevent human suffering, then it is wrong for medical researchers to keep their research confidential. C. Medical researchers should not keep information confidential if it is possible that sharing that information would prevent some unnecessary human suffering.

PRINCIPLE SUPPORT A. Incorrect. Too extreme. The arguer never claimed to "know" when human beings suffer. B. Incorrect. Stimulus never states that the most important moral principle is to prevent human suffering, so the principle does not justify the doctor's argument. C. Correct. "It is possible" matches weaker language of the stimulus.

Some scientists have expressed reservations about quantum theory because of its counterintuitive consequences. But despite rigorous attempts to show that quantum theory's predictions were inaccurate, they were shown to be accurate within the generally accepted statistical margin of error. These results, which have not been equaled by quantum theory's competitors, warrant acceptance of quantum theory. Which one of the following principles most helps to justify the reasoning above? A. A scientific theory should be accepted if it has fewer counterintuitive consequences than do its competitors. B. A scientific theory should be accepted if it has been subjected to serious attempts to disprove it and has withstood all of them.

PRINCIPLE SUPPORT A. Incorrect. We only know about resistance against attempts of disproof among competitors, not counterintuitive consequences. B. Correct. Quantum theory but not competitors have been shown accurate against attempts of disproof

Political theorist: Many people believe that the punishment of those who commit even the most heinous crimes should be mitigated to some extent if the crime was motivated by a sincere desire to achieve some larger good. Granted, some criminals with admirable motives deserve mitigated punishments. Nonetheless, judges should never mitigate punishment on the basis of motives, since motives are essentially a matter of conjecture and even vicious motives can easily be presented as altruistic. Which one of the following principles, if valid, most helps to justify the political theorist's reasoning? B. It is better to err on the side of overly severe punishment than to err on the side of overly lenient punishment. C. The legal permissibility of actions should depend on the perceivable consequences of those actions.

PRINCIPLE SUPPORT B. Correct. Supports conclusion to punish criminals regardless of intention. C. Incorrect. "Legal permissibility" is out of scope.

Principle: When none of the fully qualified candidates for a new position at Arvue Corporation currently works for that company, it should hire the candidate who would be most productive in that position. Application: Arvue should not hire Krall for the new position, because Delacruz is a candidate and is fully qualified. Which one of the following, if true, justifies the above application of the principle? A. All of the candidates are fully qualified for the new position, but none already works for Arvue. B. Of all the candidates who do not already work for Arvue, Delacruz would be the most productive for the position. C. Krall works for Arvue, but Delacruuz is the candidate who would be the most productive in the new position. D. Several candidates currently work for Arvue, but Brall and Delacruz do not. E. None of the candidates already works for Arvue, and Delacruz is the candidate who would be mmost produuctive in the new position.

PRINCIPLE SUPPORT P1: -Fully qualified candidate currently working for company --> hire most productive candidate. P2: Delacruz is a fully qualified candidate. C: -Krall. Analysis: Fully qualified candidate is not a sufficient condition. -Working at the company is a sufficient condition. A. Incorrect. Does not specify if Krall is currently working, so does not justify hiring the most productive over Krall. B. Incorrect. -Working at the company justifies hiring the most productive candidate and Delacruz being the most productive justifies hiring Delacruz. However, it does not explain why Delacruz being a fully qualified candidate is a reason for why Krall should not be hired. C. Incorrect. Does not specify if Krall is qualified, so does not justify hiring the most productive over Krall. D. Incorrect. -Working at the company justifies hiring the most productive candidate. Does not specify who is the most productive, so does not justify not hiring Krall. E. Correct. -Working at the company justifies hiring the most productive candidate. Specifics Delacruz as the most productive, justifying that Krall should not be hired.

A governmental study indicates that raising speed limits to reflect the actual average speeds of traffic on level, straight stretches of high-speed roadways reduces the accident rate. Since the actual average speed for level, straight stretches of high-speed roadways tends to be 12- kilometers per hour (75 miles per hour), that should be set as a uniform national speed limit for level, straight stretches of all such roadways. Which one of the following principles, if valid, most helps to justify the reasoning above? A. Uniform national speed limits should apply only to high-speed roadways. B. Traffic laws applying to high-speed roadways should apply uniformly across the nation. E. Any measure that reduces the rate of traffic accidents should be implemented.

PRINCIPLE SUPPORT P1: speed limits at actual average reduces accident rate on level, straight stretches of high-speed roadways. P2: actual average speed for level, straight stretches of high-speed roadways is 75 mph. C: 75 mph should be the uniform national speed for all level, straight stretches of high-speed roadways Principle: a safe procedure should be implemented. A. Incorrect. "Only" is too restrictive. B. Incorrect. "Traffic laws" does not address safety. Helps justify that it should be uniform, but does not establish whether we should set it as uniform based on the premise given. E. Correct.

Music critic: How well an underground rock group's recordings sell is no mark of that group's success as an underground group. After all, if a recording sells well, it may be because some of the music on the recording is too trendy to be authentically underground; accordingly, many underground musicians consider it desirable for a recording not to sell well. But weak sales may simply be the result of the group's incompetence. Which one of the following principles, if valid, most helps to justify the music critic's argument? A. If an underground rock group is successful as an underground group, its recordings will sell neither especially well nor especially poorly. B. An underground rock group is unsuccessful as an underground group if it is incompetent or if any of its music is too trendy to be authentically underground, or both. C. Whether an underground group's recordings meet criteria that many underground musicians consider desirable is not a mark of that group's success. D. An underground rock group is successful as an underground group if the group is competent but its recordings nonetheless do not sell well. E. For an underground rock group, competence and the creation of authentically underground music are not in themselves mark of success.

PRINCIPLE SUPPORT P: Sells well --> (maybe) too trendy to be authentically underground. -Sells well --> (maybe) incompetence. C: Sellings do not indicate an underground rock group's success. The reasoning does not define what success is. The correct principle should link incompetence and/or being too trendy to success (or lack thereof). A. Incorrect. Suggests that success depends on sellings. Furthermore, "especially" is problematic, because it is never explicitly stated at what point along the spectrum of record sales (from good seller--->poor seller) does an underground rock group go from being successful underground--> -successful underground. Furthermore, the language in this answer choice is too strong to match with the stimulus ("maybe"). B. Incorrect. Successful --> -sell well AND -sell poorly; sell well OR sell poorly --> -successful. Reinforces that sales do not reliably show success, because the meaning of high and low sales is uncertain. C. Incorrect. Does not fully justify the conclusion (links "-sell well" but not "sell well" to success). Not selling well is a desirable criteria for underground musicians, yet sellings do not indicate success. Thus meeting the desirable criteria is not a mark of success. D. Incorrect. Competent AND -sell well --> success. However, we do not want to justify "success" but rather "-success." This suggests that sales DO reflect success, which is contrary to C. E. Incorrect. Competence AND -trendy --> not marks of success. This weakens the argument. The author was trying to imply that competence and non-trendy music were important to success. And yet this answer says those factors aren't important!

Anyone believing that no individual can have an effect on society's future will as a result feel too helpless to act to change society for the better. Thus, anyone who wants to improve society should reject the belief that its future will be determined entirely by vast historical forces that individuals are powerless to change. Which one of the following principles, if valid, most helps to justify the argument? A. Anyone who believes that individuals can have an effect on society's future should act to change society for the better. B. No one who rejects the belief that society's future will be determined by vast historical forces should believe that individuals cannot have an effect on it. C. Anyone who feels too helpless to act to change society for the better should reject the belief that its future will be determined by vast historical forces that individuals are powerless to change. D. No one who wants to improve society should accept any belief that makes him or her feel too helpless to act to change society for the better. E. Each individual should act to improve society if individuals in general feel powerless in the face of vast historic forces.

PRINCIPLE SUPPORT P: -effect --> helpless to improve society; -helpless to improve society --> effect C: wants to improve society --> reject that future is determined. A: wants to improve society --> -helpless to improve society --> effect --> reject that the future is determined. We need a "should" statement that links the premise and the conclusion: "If something makes you feel helpless, then you shouldn't believe it." PREMISE: Trigger --> outcome CONCLUSION: Particular group should avoid trigger. PRINCIPLE: Particular group should avoid outcome. A. Incorrect. Illegal reversal. Effect --> improve society. B. Incorrect. Illegal reversal. Reject that future is determined --> effect. C. Incorrect. Logical flaw. Helpless to improve society --> reject that future is determined. D. Correct. Wants to improve society should not feel helpless to improve society. E. Incorrect. Not concerned with individuals "in general".

To predict that a device will be invented, one must develop a conception of the device that includes some details at least about how it will function and the consequences of its use. But clearly, then, the notion of predicting an invention is self-contradictory, for inventing means developing a detailed conception, and one cannot predict what has already taken place. Which one of the following most accurately describes the technique of reasoning employed by the argument? A. Constructing a counterexample to a general hypothesis about the future B. Appealing to definitions to infer the impossibility of a kind of occurrence C. Countering a hypothesis by indicating the falsehood of the implications of that hypothesis E. Attempting to show that predicting any event implies that it has in fact already taken place

PROCEDURE Evidence: 1. To predict a device will be invented, one must develop a conception of the device. 2. Inventing means developing a detailed conception. 3. One cannot predict what has already taken place. Conclusion: The notion of predicting an invention is self-contradictory. A. Incorrect. No counterexample offered. B. Correct. P2 defines the meaning of the invention C. Incorrect. The only implication of predicting an invention is that one must develop a conception of the device. But the argument never denies that implication. It does say that one cannot predict something that has already happened, but the argument does not claim that predicting an invention implies that one can predict what has already happened. E. Incorrect. E discusses the prediction of "any" event, yet the argument only discusses the prediction of an invention

Zachary: One would have to be blind to the reality of moral obligation to deny that people who believe a course of action to be morally obligatory for them have both the right and duty to pursue that action, and that no one else has any right to stop them from doing. Cynthia: But imagine an artist who feels morally obliged to do whatever she can to prevent works of art from being destroyed confronting a morally committed antipornography demonstrator engaged in destroying artworks he deems pornographic. According to your principle that artist has, simultaneously, both the right and the duty to stop the destruction and no right whatsoever to stop it. Which one of the following, if substituted for the scenario invoked by Cynthia, would preserve the force of her argument? A. A medical researcher who feels a moral obligation not to claim sole credit for work that was performed in part by someone else confronting another researcher who feels no such moral obligation. B. A manufacturer who feels a moral obligation to recall potentially dangerous products confronting a consumer advocate who feels morally obliged to expose product defects. C. An investment banker who believes that governments are morally obliged to regulate major industries confronting an investment banker who holds that governments have a moral obligation not to interfere with market forces. D. An architect who feels a moral obligation to design only energy-efficient buildings confronting, as a potential client, a corporation that believes its primary moral obligation is to maximize shareholder profits. E. A health inspector who feels morally obliged to enforce restrictions on the number of cats a householder may keep confronting a householder who, feeling morally obliged to keep every stray that comes along, has over twice that number of cats.

PROCEDURE (Substitution?) Z: Action is morally obligatory --> right + duty + no one else has right to stop them. C: Artist 1 feels obligation that contradicts obligation of Artist 2. This negates "no one else has right to stop them Thus...artist... Morally obliged. Right + duty. -Right. A. Incorrect. Researcher 2 feels a lack of moral obligation, rather than a contradicting obligation. B. Incorrect. Two obliged acts are not contradictory. C. Incorrect. Both have opinions about the moral obligations of government, but we don't hear about their personal moral obligations. D. Incorrect. Two obliged acts are not contradictory. E. Correct. Two obliged acts are contradictory.

Botanist: In an experiment, scientists raised domesticated radishes in a field with wild radishes, which are considered weeds. Within several generations, the wild radishes began to show the same flower color as the domesticated ones. This suggests that resistance to pesticides, which is often a genetically engineered trait, would also be passed from domesticated crop plants to their relatives that are considered weeds. Which one of the following, if true, most strengthens the botanist's argument? B. When the ratio of domesticated radishes to wild radishes in the field increased, the speed with which the flower color passed to the wild radishes also increased. E. It is more difficult for flower color to be transferred between domesticated and wild radishes than it is for almost any other trait to be passed between any two similarly related plant species.

STRENGTHEN A flawed comparison. P: in an experiment with radishes, flower color was passed from domesticated radishes to wild radishes C: resistance to pesticides would be passed from domesticated crop plants to its relatives that are considered weeds. A: since flower color passes from domesticated radishes to wild radishes, resistance to pesticides will as well. To strengthen this argument we'll look for a stronger connection between these two traits. B. Incorrect. Does not address the rate that genetically engineered traits are passed on. E. Correct. If it's more difficult for flower color to be transferred than other genetic traits, that makes it easier for resistance to pesticides to be transferred.

Psychologist: we asked 100 entrepreneurs and 100 business managers to answer various questions and rate how confident they were that their responses were correct. While members of each group were overconfident, in general the entrepreneurs were much more so than the business managers. This indicates that people who are especially overconfident are more likely to attempt to start a business in spite of the enormous odds against success than people who are less confident. Which one of the following, if true, lends the most support to the psychologist's conclusion? B. At least some of the entrepreneurs surveyed had accurately determined before attempting to start their business what the odds were against their attempts being successful. D. The business managers who were most confident were found to have attempted to start businesses in the past.

STRENGTHEN B. Incorrect. Successful assessment of odds/risk is out of scope. Overconfidence doesn't care about accurate estimations; it just relies on actions that don't align with the subjective risks viewed. D. Correct. Presents more examples of the correlation (overconfidence and entrepreneurship)

A small collection of copper-alloy kitchen implements was found in an abandoned Roman-era well. Beneath them was a cache of coins, some of which dated to 375 A.D. The implements, therefore, were dropped into the well no earlier than 375 A.D. Which one of the following, if true, most strengthens the argument? A. The coins used in the Roman Empire often remained in circulation for many decades. B. The coins were found in a dense cluster that could not have been formed by coins slipping through an accumulation of larger objects. E. Items of jewelry found beneath the coins were probably made around 300 A.D.

STRENGTHEN P: implements found above coins from 375 A.D. C: implements were dropped no earlier than 375 A.D. A: since the coins are located at the bottom of the well, they must have been dropped there first A. Incorrect. Does not matter how long the coins remained in circulation. The presence of one 375 A.D. coin indicates that it was dropped at least no earlier than 375 A.D. B. Correct. Matches the assumption. C. Incorrect. A premise booster--does not fully address the gap between the premise and conclusion. is very tempting because it's a premise booster. Suggests that the jewelry below the coins is also older than the coins. However, does not suggest that the coins below the implements are also older than the implements.

An economist has argued that consumers often benefit when government permits a corporation to obtain a monopoly. Without competition, a corporation can raise prices without spending nearly as much on advertising. The corporation can then invest the extra money in expensive research or industrial infrastructure that it could not otherwise afford, passing the fruits of these investments on to consumers. Which one of the following, if true, most strengthens the economist's argument? A. The benefits to consumers are typically greater if a corporation invests in expensive research or industrial infrastructure than if the corporation spends the same amount of money in another way. B. The government's permitting a corporation to obtain a monopoly is advantageous for consumers only if that corporation passes the fruits of at least some of its investments to consumers. C. If a corporation obtains a monopoly, the disadvantage to consumers of any higher prices will be outweighed by the advantages from extra investments in expensive research or industrial infrastructure made by that corporation.

STRENGTHEN P: monopolies raise and invest extra $ in research and infra that benefit consumers C: monopolies often benefit consumers A: the benefits to consumers will outweigh the costs of monopolies raising high prices A. Incorrect. Refers to corporations, so does not strengthen C about monopolies specifically. B. Incorrect. Does not strengthen by makes the condition for monopolies benefiting consumers more specific. C. Correct. Directly addresses assumption.

Physics professor: some scientists claim that superheated plasma in which electrical resistance fails is a factor in causing so-called "ball lighting." If this were so, then such lighting would emit intense light and, since plasma has gaslike properties, would rise in the air. However, the instances of ball lighting that I observed were of low intensity and floated horizontally before vanishing. Thus, superheated plasma with failed electrical resistance is never a factor in causing ball lighting. The physics professor's conclusion follows logically if which one of the following is assumed? A. Superheated plasma in which electrical resistance fails does not cause types of lighting other than ball lighting. B. The phenomena observed by the physics professor were each observed by at least one other person. C. Ball lighting can occur as the result of several different factors. D. Superheating of gaslike substances causes bright light to be emitted. E. All types of ball lighting have the same cause.

SUFFICIENT ASSUMPTION A. Incorrect. Unclear and irrelevant bearing on C. B. Incorrect. Unclear and irrelevant bearing on C. C. Incorrect. Unclear bearing, but if anything weakens by suggesting that there exist numerous causes for confusion. D. Incorrect. Premise booster. E. Correct. Affirms that the observed instance of ball lighting is representative of all instances.

Secondary school students achieve broad mastery of the curriculum if they are taught with methods appropriate to their learning styles and they devote significant effort to their studies. Thus, if such broad mastery is not achieved by the students in a particular secondary school, those students are not being taught with methods appropriate to their learning styles. The conclusion can be properly drawn if which one of the following is assumed? A. As long as secondary school students are taught with methods appropriate to their learning styles, they will devote significant effort to their studies. B. Even if secondary school students are taught with methods appropriate to their learning styles, they will not achieve broad mastery of the curriculum if they do not devote significant effort to their studies. C. Secondary school students do not achieve broad mastery of the curriculum if they are not taught with methods appropriate to their learning styles. D. Teaching secondary school students with methods appropriate to their learning styles does not always result in broad mastery of the curriculum by those students. E. Secondary school students who devote significant effort to their studies do not always achieve broad mastery of the curriculum.

SUFFICIENT ASSUMPTION P1: Appropriate methods AND significant effort --> mastery; -mastery --> -appropriate methods OR -significant effort P2: -Mastery. C: -Appropriate methods A: C states that whenever -mastery it occurs, it always requires -appropriate methods, no matter which half of the OR consequence occurs. Our two options are "-mastery --> -appropriate methods" or "-mastery --> -appropriate methods AND -significant effort." To justify this, our assumption must be that -significant effort --> -appropriate method. A. Correct. Appropriate methods --> significant effort; -significant effort --> -appropriate method. Establishes that no matter which half of the OR consequence occurs, -appropriate methods also occurs. B. Incorrect. Partial illegal reversal. Appropriate methods AND -significant effort --> -mastery. C. Incorrect. Illegal reversal. -Appropriate method --> -mastery. D. Incorrect. Already implied by the stimulus, since appropriate methods are only sufficient if they are accompanied by effort. E. Incorrect. Same reasoning as D.

Whoever murdered Jansen was undoubtedly in Jansen's office on the day of the murder, and both S and H were in J's office that day. If H had committed the murder, the police would have found either his fingerprints or his footprints at the scene of the crime. But if S was the murderer, she would have avoided leaving behind footprints or fingerprints. The police find fingerprints but no footprints at the scene of the crime. Since the fingerprints were not H's he is not the murderer. Thus S must be the killer. Which one of the following, if assumed, allows the conclusion that S was the killer to be properly inferred? A. If there had been footprints at the scene of the crime, the police would have found them. B. J's office was the scene of the crime. C. No one but H and S was in J's office on the day of the murder. D. The fingerprints found at the scene of the crime were not J's. E. The fingerprints found at the scene of the crime were not S's.

SUFFICIENT ASSUMPTION P1: the killer was in J's office; both S and H were in J's office. P2: H --> fingerprints OR footprints P3: S --> -fingerprints or -footprints (It could be the case that she avoided leaving both. Contrapositive: fingerprints and footprints --> -S) P4: the police found fingerprints but no footprints. The fingerprints were not H's P5: -H A1: The sufficient condition fingerprints and footprints --> -S is not met, so we cannot conclude that S is not guilty) A2: -H --> S...assumes that there are only two options. C: S is the killer *Remember that -fingerprints or -footprints is necessary, NOT sufficient for S being the killer A. Incorrect. Necessary but not sufficient. B. Incorrect. "Scene of crime" is consistent in premises and conclusion. Does not necessarily need to be J's office. C. Correct. Allows -H --> S. D. Incorrect. Does not influence logic. E. Incorrect. If anything, weakens C that S committed the murder because if the fingerprints found at the scene of the crime were not S's, then applying the same justification the argument uses to dismiss Herbert as the Killer we could dismiss Samantha as the killer

If a wig has any handmade components, it is more expensive than one with none. Similarly, a made-to-measure wig ranges from medium-priced to expensive. Handmade foundations are never found on wigs that do not use human hair. Furthermore, any wig that contains human hair should be dry-cleaned. So all made-to-measure wigs should be dry-cleaned. The conclusion of the argument follows logically if which one of the following is assumed? A. Any wig whose price falls in the medium-priced to expensive range has a handmade foundation. B. If a wig's foundation is handmade, then it is more expensive than one whose foundation is not handmade. C. A wig that has any handmade components should be dry-cleaned. D. If a wig's foundation is handmade, then its price is at least in medium range. E. Any wig that should be dry-cleaned has a foundation that is handmade.

SUFFICIENT ASSUMPTION P: Handmade components --> expensive. Made-to-measure --> medium-priced to expensive. Handmade foundation --> human hair. Human hair --> dry-cleaned. C: Made-to-measure --> dry-cleaned. A: Medium-priced to expensive --> human hair to allow made-to-measure --> medium-priced to expensive --> human hair --> dry-cleaned. OR Medium-priced to expensive --> handmade foundation to allow made-to-measure --> medium-priced to expensive --> handmade foundation --> human hair --> dry-cleaned. A. Correct. Medium-priced to expensive --> handmade foundation. B. Incorrect. "More expensive" is out of scope. C. Incorrect. Handmade --> dry-cleaned, yet does not connect made-to-measure --> dry-cleaned. D. Incorrect. Illegal reversal of desired assumption: handmade --> medium-priced to expensive. E. Incorrect. Illegal reversal of C: dry-cleaned --> handmade.

Cecile's association requires public disclosure of an officer's investments in two cases only: when an officer is authorized to disburse association funds, and when an officer sits on the board of a petrochemical company. Cecile, an officer who is not authorized to disburse funds, sits on the board of just one company, a small timer business. Therefore, there is no reason for Cecile to publicly disclose her investments at this time. The conclusion of the argument follows logically if which one of the following is assumed? A. Cecile will not be appointed to a position in the association that authorizes her to disburse funds. C. The association requirements provide the only reasons there might be for Cecile to disclose her investments. D. The timber business on whose board Cecile sits is owned by a petrochemical company.

SUFFICIENT ASSUMPTION P: (1) authorized to disburse --> public disclosure required; board of a petrochemical company --> public disclosure required. (2) Cecile is not authorized to disburse. (3) Cecile on the board of a timber business. C: no reason for Cecile to publically disclose currently. A: gap between the requirement to disclose in the premises and reason to disclose in C. A. Incorrect. C is based on "this time," so future descriptions are out of scope. C. Correct. Tempting to eliminate because the stimulus describes "two cases only." However, the stimulus describes requirements while C describes the reasons for public disclosure, which may be more numerous. D. Incorrect. No bearing on C; already given that Cecile sits on the "board of a timer business" and thus -board of a petrochemical company. If anything, implies board of a petrochemical company, thus weakening C by satisfying the conditional board of a petrochemical company --> public disclosure.

Historian: it is unlikely that someone would see history as the working out of moral themes unless he or she held clear and unambiguous moral beliefs. However, one's inclination to morally judge human behavior decreases as one's knowledge of history increases. Consequentially, the more history a person knows, the less likely that person is to view history as the working out of moral themes. The conclusion of the argument is properly drawn if which one of the following is assumed? A. Historical events that fail to elicit moral disapproval are generally not considered to exemplify a moral theme. B. The less inclined one is to morally judge human behavior, the less likely it is that one holds clear and unambiguous moral beliefs. C. Only those who do not understand human history attribute moral significance to historical events. D. The more clear and unambiguous one's moral beliefs, the more likely one is to view history as the working out of moral themes. E. People tend to be less objective regarding a subject about which they possess extensive knowledge than regarding a subject about which they do not possess extensive knowledge.

SUFFICIENT ASSUMPTION P: -Clear and unambiguous beliefs --> -history = working of moral themes. As knowledge of history increases, moral judgment decreases. C: As knowledge of history increases, history = working of moral themes decreases. A: history increases --> moral judgment decreases --> clear and unambiguous moral beliefs decrease --> history = working of moral themes decreases. A. Incorrect. "Moral disapproval" is out of scope. B. Correct. Moral judgment decreases --> clear and unambiguous moral beliefs decrease. D. Incorrect. Illegal reversal. Clear and unambiguous beliefs --> history = working of moral themes. E. Incorrect. "Objective" is out of scope.

Psychologist: psychotherapists who attempt to provide psychotherapy on radio or television talk shows are expected to do so in ways that entertain a broad audience. However, satisfying this demand is nearly always incompatible with providing high-quality psychological help. For this reason, psychotherapists should never provide psychotherapy on talk shows. Which one of the following principles must be assumed in order for the psychologist's conclusion to be properly drawn? C. Psychotherapy should never be provided in a context in which there is any chance that the therapy might be of less than high quality. E. Psychotherapists should never attempt to provide psychological help in a manner that makes it unlikely to be of high quality.

SUFFICIENT ASSUMPTION P: psychotherapy on talk shows will almost always result in not high-quality therapy. C: psychotherapists should never provide psychotherapy on talk shows A: if psychotherapists provide psychotherapy in a manner that may result in it not being high-quality, the psychotherapists should not provide psychotherapy in that manner C. Incorrect. Too extreme and addresses all forms of psychotherapy but not psychotherapy given by psychotherapists specifically. This unnecessarily restricts an amateur giving potentially not high-quality psychotherapy to her friend. E. Correct. Addresses gap.

Vaccine question

The second sentence, which begins with "This goal will be accomplished" closes the gap between the government's policy and the means to get there via vaccinations. The remaining issue/gap is whether it'll be necessary for the high-risk people to get a vaccine for a different strain every year, i.e. (D).

In 1955, legislation in a certain country gave the government increased control over industrial workplace safety conditions. Among the high-risk industries in that country, the likelihood that a worker will suffer a serious injury has decreased since 1955. The legislation, therefore, has increased overall worker safety within high-risk industries. Which one of the following, if true, most weakens the argument above? A. Because of technological innovation, most workplaces in the high-risk industries do not require as much unprotected interaction between workers and heavy machinery as they did in 1955. D. The number of work-related injuries occurring within industries not considered high-risk has increased annually since 1955.

WEAKEN A. Correct. Suggests alternative causes for the decline D. Incorrect. The conclusion is about the likelihood that a worker will be involved in an accident, whereas C is about the number of accidents. Be cautious of percent v. total amount arguments. Furthermore, the conclusion addresses high-risk industries, whereas C addresses all industries.

Company spokesperson: In lieu of redesigning our plants, our company recently launched an environmental protection campaign to buy and dispose of old cars, which are generally highly pollutive. Our plants account for just 4% the local air pollution, while automobiles that predate 1980 account for 30%. Clearly, we will reduce air pollution more by buying old cars than we would by redesigning our plants. Which one of the following, if true, most seriously weakens the company spokesperson's argument? A. Only 1% of the automobiles driven in the local area predate 1980. C. Because the company pays only scrap metal prices for used cars, almost none of the cars sold to the company still run. D. Automobiles made after 1980 account for over 30% of local air pollution.

WEAKEN A. Incorrect. The 1% of automobiles that predate 1980 is irrelevant, because it is already given that these automobiles account for 30% of air pollution. C. Correct. Suggests that the company will not be decreasing the number of running cars (and thus pollution emitted) because it only purchases non-running cars. D. Incorrect. Does not change the fact that replacing automobiles that predate 1980 will reduce a higher percentage of air pollution than redesigning the plant.

Computer manufacturers and retailers tell us that the complexity involved in connecting the various components of personal computers is not a widespread obstacle to their use, but this is wrong. Customers who install accessories to their personal computers have to take full responsibility for the setting of jumpers and switches to satisfy mysterious specifications. Many accessories require extra software that can cause other accessories to stop working; adding a modem, for instance, may disable a printer. Which one of the following, if true, most seriously weakens the argument? A. Personal computer instruction manuals usually explain the purposes of the jumpers and switches. D. A personal computer is usually sold as part of a package that includes accessories and free installation.

WEAKEN P1: customers must take responsibility for complicated accessories P2: many accessories require extra software that causes other accessories to stop working C: incorrect that complexity of personal computers prevents widespread use of personal computers A. INCORRECT. Even if the manuals explain the purposes of the jumpers and switches, does not explain complexity D. CORRECT

Nightbird is an unsigned painting that some attribute to the celebrated artist Larocque. Experts agree that it was painted in a style indistinguishable from that of Larocque and that if it was not painted by Larocque, it was undoubtably painted by one of his students. A recent analysis showed that the painting contains orpiment, a pigment never yet found in a work attributed to Larocque. Therefore, the painting must have been done by one of Larocque's students. Which one of the following, if true, most weakens the argument? A. Few of Larocque's students ever used painting techniques that differed from Larocque C. No painting currently recognized as the work of one of Larocque's students contains orpiment

WEAKEN P1: if not painted by Larocque, painted his student P2: painting contains orpiment, a pigment never yet found in a work attributed to Larocque C: painted by student A: if the painting does not resemble Larocque's existing work, then it was not painted by Larocque A. Incorrect. "Few" is not strong enough. The one student whose technique differed from Larocque's may have painted the piece. C. Correct. Under the argument's assumption, Larocque's students also could not have painted the piece. Makes it equally unlikely that a student painted it.

Biologists have noted reproductive abnormalities in fish that are immediately downstream of paper mills. One possible cause is dioxin, which paper mills release daily and which can alter the concentration of hormones in fish. However, dioxin is unlikely to be the cause, since the fish recover normal hormone concentrations relatively quickly during occasional mill shutdowns and dioxin decomposes very slowly in the environment. Which one of the following statements, if true, most seriously weakens the argument? B. The rate at which dioxin decomposes varies depending on the conditions to which it is exposed. C. Normal river currents carry the dioxin present in the river far downstream in a few hours.

WEAKEN P1: mills release dioxin, and dioxin decomposes very slowly in the environment P2: when mills shut down, fish recover normal hormone concentrations C: dioxin not likely the cause of hormone changes and reproductive abnormalities in fish immediately downstream of mills *attempt to disprove the cause and effect relationship by demonstrating cause without effect B. Incorrect. It does not matter if there is this variability in other environments; dioxin decomposes very slowly in this environment. C. Correct. Normal river currents carry the dioxin further downstream, so dioxin is likely not affecting these fish; as a result, we cannot claim that dioxin is not the cause because the recovery is based on a decrease in dioxin *here, there is no cause and also no effect; however, this does not dispute the relationship

Book publishers have traditionally published a few books that they thought were of intrinsic merit even though these books were unlikely to make a profit. Nowadays, however, fewer of these books are being published. It seems, therefore, that publishers now, more than ever, are more interested in making money than in publishing books of intrinsic value. Which one of the following statements, if true, most seriously weakens the argument? B. There has been a notable decline in the quality of books written in recent years. C. In the past, often books of intrinsic value would unexpectedly make a sizable profit. E. In recent years, profits in the book publishing industry have been declining.

WEAKEN P1: previously, publish books with intrinsic value and unlikely to make profit P2: decline in publishing these books C: increase in concern for profit B. Correct. Suggests an alternative reason besides concern for profit that describes decline in publishing these books. C. Incorrect. Suggests that previous publication was motivated by profit. However, does not influence whether the company nowadays is MORE interested in making profits; this is unknown. E. Incorrect. Offers an explanation for why there's a decline in the number of those books that don't make a profit, but seems to suggest the same explanation (profit) as was concluded in the argument

A cup of raw milk, after being heated in a microwave oven to 50 degrees Celsius, contains half its initial concentration of a particular enzyme, lysozyme. If, however, the milk reaches that temperature through exposure to a conventional heat source of 50 degrees Celsius, it will contain nearly all of its initial concentration of enzyme. Therefore, what destroys the enzyme is not beat but microwaves, which generate heat. Which one of the following, if true, most seriously weakens the argument? A. Heating raw milk in a microwave oven to a temperature of 100 degrees Celsius destroys nearly all of the lysozyme initially present in that milk. B. Enzymes in raw milk that are destroyed through excessive heating can be replaced by adding enzymes that have been extracted from other sources. C. A liquid exposed to a conventional heat source of exactly 50 degrees Celsius will reach that temperature more slowly than it would if it were exposed to a conventional heat source hotter than 50 degrees. D. Milk that has been heated in a microwave oven does not taste noticeably different from milk that has been briefly heated by exposure to a conventional heat source. E. Heating any liquid by microwave creates small zones within it that are much hotter than the overall temperature that the liquid will ultimately reach.

WEAKEN P: (1) Microwave to 50 degrees: 1/2 enzyme (2) A conventional heat source to 50 degrees: all enzyme C: Microwave, not heat, destroys enzyme A. Incorrect. Does not weaken; consistent with existing evidence. B. Incorrect. Replacement is out of scope. C. Incorrect. Speed of heat source has unclear bearing on C; furthermore, no comparison to microwave, so incorrect. D. Incorrect. Taste is out of scope. E. Correct. Suggests that temperature is inconsistent, thus weakening C that microwaves, rather than heat, destroys enzymes. Furthermore, offers an alternate cause of excess heat rather than the microwave that destroys enzymes. *Weakens the causation flaw by offering a potential alternative cause!

The Kiffer Forest Preserve, in the northernmost part of the Abbimac Valley, is where most of the bears in the valley reside. During the eight years that the main road through the preserve's bear population has nearly doubled. Thus, the valley's bear population will increase if the road is kept closed. Which one fo the following, if true, most undermines the argument? A. Most of the increase in the preserve's bear population over the past eight years is due to migration B. Only some of the increase in the preserve's bear population over the past eight years is due to migration of bears from other parts fo the Abbimac Valley. C. Only some of the increase in the preserve's bear population over the past eight years is due to migration of bears from outside the Abbimac Valley. D. The bear population in areas of the Abbimac Valley outside of the Kiffer Forest Preserve has decreased over the past eight years. E. The bear population in the Abbimac Valley has remained about the same over the past eight years.

WEAKEN P: Before roadblock, most bears in Kiffer Forest Preserve. After roadblock, bear population in Kiffer Forest Preserve doubled. C: If we keep the road closed, bear population in the valley will increase. Analysis: Note the term shift : P describes population increase in the preserve, but C describes population in the entire valley. Akin to: there are more rich people in NY now than there were ten years ago because of the new tax breaks, so if we continue the tax breaks, there will be more rich people in the world. Overlooks the possibility that rich people just moved to NY and the number of rich people in the world will not change. A: Population increase in preserve --> population increase in valley. A. Incorrect. Explains why the population of the preserve increased, but does not weaken/suggest that migration will not continue. B. Incorrect. Does not weaken claim that population increase will continue. May strengthen; could counter the objection of "what if all the growth in population in the preserve just comes from other parts of the valley?". C. Incorrect. Same reasoning as B. D. Incorrect. Would weaken if it was given that the bear population increased due to migration. However, bear population may still increase via other methods besides migration. Furthermore, it could be that the decrease does not match the increase and population increase in the preserve still occurs. For example, perhaps the general valley outside of the preserve lost 2 bears overall, while the preserve gained 80 (for example due to higher birth rate). The conclusion could still hold. E. Correct. Population increase in preserve but not in valley. Suggests that migration caused increase in preserve, and that the observation of population increase in the preserve will not cause population increase in the valley.

High school students who feel that they are not succeeding in school often drop out before graduating and go to work. Last year, however, the city's high school dropout rate was significantly lower than the previous year's rate. This is encouraging evidence that the program instituted two years ago to improve the moral of high school students has begun to take effect to reduce dropouts. Which one of the following, if true about the last year, most seriously weakens the argument? A. There was a recession that caused a high level of unemployment in the city. D. High schools in the city established placement offices to assist their graduates in obtaining employment.

WEAKEN P: Some high school students drop out and go to work. Last year the dropout rate was lower than the previous year. C: The morale program is reducing the number of dropouts. A. Correct. Suggests an alternative cause for the lower dropout rate. If unemployment was high then students would not have been able to drop out and find jobs. D. Incorrect. Only affects those graduated and so doesn't offer an alternative reason to stay in school. Furthermore, we cannot assume that the jobs available to graduates are more enticing than dropping out to work.

Poetry journal patron: Everybody who publishes in The Brick Wall has to agree in advance that if a poem is printed in one of its regular issues, the magazine also has the right to reprint it, without monetary compensation, in its annual anthology. The Brick Wall makes enough money from sales of its anthologies to cover most operating expenses. So if your magazine also published an anthology of poems first printed in your magazine, you could depend less on donations. After all, most poems published in your magazine are very similar to those published in The Brick Wall. Which one of the following, if true, most seriously weakens the argument above? A. Neither The Brick Wall nor the other magazines under discussion depends on donations to cover most operating expenses. B. Many of the poets whose work appears in The Brick Wall have had several poems rejected for publication by the other magazine in discussion. E. The Brick Wall's annual poetry anthology always contains a number of poems by famous poets not published in the regular issues of the magazine.

WEAKEN P: The Brick Wall makes enough $ from anthology sales to cover most operating expenses. Most poems published in your magazine are very similar to The Brick Wall's. C: If you published an anthology, you could depend less on donations A: The two magazines are similar enough that what happens to The Brick Wall should also happen to the magazine. Argument based on analogy. To weaken, point out relevant differences. A. Incorrect. A similarity. Furthermore, so what if the magazine in question does not depend on donations to cover MOST operating expenses? The conclusion merely says that selling an anthology would LESSEN dependence on donations (even if it's just by 1%). B. Incorrect. "Many" is a weak quantity. The evidence only claimed that "MOST poems in the two mags are similar", so that leaves plenty of room for the exceptions that (B) describes. E. Correct. Points out a significant difference.

We can learn about the living conditions of a vanished culture by examining its language. Thus, it is likely that the people who spoke Proto-Indo-European, the language from which all Indo-European languages descended, lived in a cold climate, isolated from the ocean or sea, because Proto-Indo-European lacks a word for "sea," yet contains words for "winter," "snow," and "wolf." Which one of the following, if true, most seriously weakens the argument? A. A word meaning "fish" was sued by the people who spoke Proto-Indo-European. Incorrect. "Fish" does not mandate oceans or lakes; may occur in lakes or rivers. B. Some languages lack words for prominent elements of the environments of their speakers. Correct. If some languages lack words for prominent features of the speakers' environment, then it is possible that the Proto-Indo-Europeans did live by the sea and lacked a word for it. C. There are no known languages today that lack a word for "sea." Incorrect. Present state of languages does not affect the past relationship between the environment and Proto-Indo-European. If anything, weakens the premise that a language could lack a word for "sea," yet does not affect reasoning. D. Proto-Indo-European possess words for "heat." Incorrect. No influence on reasoning. E. The people who spoke Proto-Indo-European were nomadic. Incorrect. No influence on reasoning. Even if they were nomadic, they may still have lived in a cold climate isolated from the ocean.

WEAKEN P: (1) learn about living conditions through language. (2) Proto-Indo-European lacks the word for "sea" yet contains words for "winter," "snow," and "wolf." C: Proto-Indo-European people lived in a cold climate isolated from the ocean or sea. A. Incorrect. "Fish" does not mandate oceans or lakes; may occur in lakes or rivers. B. Correct. If some languages lack words for prominent features of the speakers' environment, then it is possible that the Proto-Indo-Europeans did live by the sea and lacked a word for it. C. Incorrect. Present state of languages does not affect the past relationship between the environment and Proto-Indo-European. If anything, weakens the premise that a language could lack a word for "sea," yet does not affect reasoning. D. Incorrect. No influence on reasoning. E. Incorrect. No influence on reasoning. Even if they were nomadic, they may still have lived in a cold climate isolated from the ocean.

Today's farmers plant only a handful of different strains of a given crop. Crops lack the diversity that they had only a few generations ago. Hence, a disease that strikes only a few strains of crops, and that would have had only minor impact on the food supply in the past, would devastate it today. Which one of the following, if true, would most weaken the argument? A. In the past, crop diseases would often devastate food supplies throughout entire regions. B. Affected crops can quickly be replaced from seed banks that store many strains of crops. E. Today's crops are much less vulnerable to damage from insects or encroachment by weeds than were crops of a few generations ago.

WEAKEN P: crops lack diversity today C: disease that affects only a few strains of crops would devastate food supply today A. Incorrect. Although a comparison is made between past and present, the devastation of the past does not influence the reasoning regarding the devastation of today. B. Correct. Presents a solution to the lack of diversity that weakens reasoning. E. Incorrect. The discussion of factors besides disease are out of scope. However, if anything, weakens.

Linguist: you philosophers say that we linguists do not have a deep understanding of language, but you have provided no evidence. Philosopher: well, you have said that you believe that "Joan and Ivan are siblings" is identical in meaning to "Ivan and Joan are siblings." But this cannot be the case, for the sentences are physically different; yet for two things to be identical, they must have all the same attributes. Of the following, which one is the strongest logical counter that the linguist can make to the philosopher? A. Two things can have a few minor differences and still be identical. C. It is necessarily true that Joan is Ivan's sibling if Ivan is Joan's sibling. D. The issue is not whether the two sentences are completely identical, but whether they mean the same thing.

WEAKEN Philosopher: P: the sentences are physically different + being identical requires that two things have all the same attributes C: "J and I are sibs" is NOT identical in meaning to "I and J are sibs" A: "Identical in meaning" and "identical" are the same. A. Incorrect. By definition, if two things have any differences, they aren't identical. Does not counter the philosopher's argument. Weakens the premise, but does not show how you can accept the premise without being forced to derive the conclusion. C. Incorrect. Does not address the physical differences between the sentences or the "identical meaning"/"identical" distinction. Weakens the conclusion, but does not weaken the reasoning. D. Correct. Questions the assumption of "identical in meaning" and "identical"

Scientists have shown that older bees, which usually forage outside the hive for food, tend to have larger brains than do younger bees, which usually do not forage but instead remain in the hive to tend to newly hatched bees. Since foraging requires greater cognitive ability than does tending to newly hatched bees, it appears that foraging leads to the increased brain size of older bees. Which one of the following, if true, most seriously weakens the argument above? A. Bees that have foraged for a long time do not have significantly larger brains than do bees that have foraged for a shorter time. E. The brains of older bees that never leaned to forage are the same size as those of their foraging counterparts of the same age.

WEAKEN P: Older bees generally have bigger brains than younger bees. Older bees do more foraging and less tending to babies than younger bees. Foraging requires more cognition than tending to babies. Conclusion: Foraging causes the increased brain size of older bees. Correlation v causation flaw; to weaken, demonstrate... 1. no relationship between x and y 2. reversed direction of causality 3. third variable A. The author is only arguing that foraging leads to older bees' bigger brains (compared to young bees). The author doesn't need to assume that the longer you forage, the bigger your brain would be to still be correct. Furthermore, this answer choice still does not give us an indication that foraging is increasing brain size otherwise. E. Correct. Controls for the effect of age. Demonstrates that foraging does not increase brain size.


Kaugnay na mga set ng pag-aaral

**chem unit 1 practice problems v2

View Set

NYSTCE CST Multisubject Part 1 (241)

View Set

5A - Financial and Nonfinancial Measures of Performance Management

View Set

Ch. 37: Child with a Communicable Disease

View Set

Unit 12 Interest Groups Key Terms and Quiz

View Set